Вы находитесь на странице: 1из 83

PROFESSIONAL RESPONSIBILITY MPRE OUTLINE (2007)

Professor Jeffrey S. Kinsler1

Copyright (2007). All rights reserved.

Introduction
The ABAs Model Rules of Professional Conduct (Rules) prescribe the standards to which a lawyer must adhere (or, in some cases, aspire) to avoid being subject to discipline by the state bar. The easiest way to understand the Rules is to consider them in connection with the myriad of people to whom a lawyer owes duties. A lawyers primary duty, of course, is to his or her client. But, depending on the circumstances, a lawyer may owe duties -- sometimes conflicting duties -- to others, including: 1. 2. 3. 4. 5. 6. 7. 8. a court or other tribunal opposing counsel third parties (witnesses, etc.) government agencies the public at large (e.g., pro bono or advertising) former clients other members of the lawyers firm the profession

Each of these relationships is examined in this outline. This is followed by a survey of a judges duties under the ABAs Model Code of Judicial Conduct. Law students should also have a general understanding of the torts of legal malpractice, fraud, and breach of fiduciary duty, Federal Rules of Civil Procedure 11 and 37, and at least a basic understanding of such crimes as obstruction of justice. Each of these topics is lightly examined in this outline.

THE DUTIES OF A LAWYER Each state regulates its own lawyers, and each federal court regulates the lawyers that practice before it. At the state level, the admission and regulation of lawyers is generally performed by a combination of state legislatures and state supreme courts. Many states, such as North Carolina, have integrated state bar associations in which all attorneys licensed in the state must be members. In most jurisdictions, the state bar association is the initial disciplinary authority for lawyers; it receives and hears complaints against lawyers. State bar disciplinary proceedings are not criminal prosecutions, but lawyers subject to such proceedings are entitled to certain constitutional protections, such as the right (without penalty) to assert the 5th Amendments privilege against self-incrimination and the right to confront and crossexam witnesses. State bar associations have the power to impose discipline on lawyers ranging from permanent disbarment to private censure. Appeals are generally heard by the states supreme court (in most jurisdictions, appeals may only be taken by the lawyer; if the complaint is dismissed, the complainant may not appeal). A lawyer who practices law in a jurisdiction in which he or she is not licensed or appears in a state or federal court in which he or she is not admitted to practice (permanently or pro hac vice) is subject to discipline for the unauthorized practice of law. I. Lawyers Duty to Client

In addition to the Model Rules of Professional Conduct, a lawyers duties to a client are governed by the laws of contract, agency, and fiduciary relationships. Contract law, for example, governs the scope of the lawyer-client relationship and the fee agreements between a lawyer and client. In addition, the law of agency prescribes the duties a lawyer owes a client, including the duties of care, loyalty, and good faith and fair dealing. A. A lawyer must represent a client competently 1. competent representation requires the legal knowledge, skill, thoroughness, and preparation reasonably required; generally, the required proficiency is that of a general practitioner Example: L represented A in the case of A v. B. B prevailed at the trial court. Under the applicable rules, A has 30 days to file a notice of appeal. Because of a vacation and a busy schedule, L failed to comply with the 30-day deadline and A is now barred from pursuing an appeal. If the appeal had been filed, A would have prevailed. L is subject to discipline and subject to civil liability (i.e., malpractice). 2. a lawyer may (but is usually not required to) consult with other lawyers/experts to acquire the requisite competency

a. note: fee-splitting with attorneys outside the lawyers firm requires written consent of the client (infra). b. a lawyer may retain a consulting expert in a field other than law to assist the lawyer in representing a client; a consulting experts opinions, etc. are generally protected from disclosure under Federal Rule of Civil Procedure (FRCP) 26; the consulting expert can sit at counsel table, attend depositions, attend settlement negotiations, etc. c. a lawyer may be required to consult with another lawyer if the matter calls for some specific expertise, such as patent law or admiralty, that the ordinary lawyer does not possess

3. a lawyer may acquire expertise (e.g., by studying, researching) in a particular field in order to represent a client Example: client asks her friend, a criminal defense lawyer, to draft a will for client; although attorney has no experience drafting wills, the attorney may acquire such expertise through necessary study to handle the clients case Example: A has a difficult tax problem. A visits her former college roommate, L, and asks L to represent her. L is a domestic relations lawyer and has never handled a tax problem. A insists that L take the case. May L take the case? Yes, if A undertakes a study of tax law necessary to competently handle the matter. 4. in emergencies, a lawyer may give advice or assistance on issues on which the lawyer is not fully competent, but the lawyer should refer the client to a competent lawyer or associate with a competent lawyer as soon as practical; the lawyer should also limit emergency advice to that reasonably necessary Example: a tax lawyer may assist a client with bail for a DUI at 2:00 a.m. when no other lawyers are available, but should refer the case to another lawyer as soon as practical. If the tax lawyer mishandles the bail issue, the lawyer will probably not be disciplined because of the emergency nature of the matter. 5. a lawyer should engage in continuing study and education of the law, even if not compulsory; formal CLE, however, is not required by the Rules, but is required by most state bars. MPRE Question 1

6. a lawyer must decline representation if the lawyer believes that she is not (and cannot become) competent to effectively represent the client 7. if a lawyer fails to competently represent a client and the client suffers damages as a result thereof, the lawyer is subject not only to discipline but also may be subject to civil liability (i.e., malpractice, fraud, and, of course, intentional torts). The elements of negligence apply to legal malpractice actions. A lawyer is expected to exercise a duty of care similar to that of an ordinary, prudent general practitioner; a lawyer who holds herself out as a specialist, such as a patent lawyer, may be subject to a higher standard. The Rules are not to be used by courts to establish the applicable standard of care, but may be used in malpractice cases as relevant evidence of a breach of the standard of care. 8. B. The Rules do not require malpractice insurance, but such insurance is advisable.

A lawyer must allow the client to make crucial decisions (with consultation): 1. in civil cases, the client decides whether to sue and whether to 2. in criminal cases, the client decides what plea to enter (e.g., guilty or innocent), whether to waive a jury trial, whether to testify, and whether to appeal 3. tactical (e.g., whether to depose a witness) and technical (e.g., whether to consent to an extension to file discovery responses) are decisions generally made by the lawyer, after consultation with the client (where possible) a. generally, if a client objects to a lawyers tactical or technical decisions, the lawyer should defer to the clients wishes, unless the clients wishes are contrary to the Rules or the law

settle

4. if a clients decision-making ability is impaired (i.e., mental incompetence, underage, etc.), the lawyer shall try to maintain a normal lawyer-client relationship to the maximum extent possible and treat the client with the same attention and respect as all other clients a. if a guardian has been appointed, the guardian is authorized to make decisions for the ward, but

the lawyer shall continue to treat the ward with attention and respect b. in extreme cases, the lawyer may seek the appointment of a guardian (or other appropriate action, such as referring a client for evaluation), but only if the lawyer reasonably believes the client cannot adequately act in the clients own best interests. In such cases, the lawyer is impliedly authorized to disclose confidential client information to the extent necessary to protect the clients interests. in an emergency (where the clients health, property or finances are at risk and the client has no other representative), a lawyer may take steps to protect an impaired client even if the client is unable to establish an attorney-client relationship; the lawyer should only take such steps as are necessary to maintain the status quo, and normally should not seek compensation for such representation

c.

C.

A lawyer and client may agree to limit the scope of the lawyers representation (e.g., the lawyer will handle the trial, but not any appeals) if the limitation is reasonable under the circumstances, except the lawyer may not: 1. limit the right of the client to fire the lawyer 2. ask the client to accept incompetent representation (e.g., waive claims for ordinary negligence) 3. 4. limit the clients right to settle the case prospectively limit the lawyers liability for malpractice Example: Ls standard retainer agreement contains the following clause: Client agrees not to hold Lawyer liable for negligence and agrees not to file suit against Lawyer. L is subject to discipline.

D.

A lawyer may not knowingly assist a client in fraudulent or illegal conduct or advise a client on how to violate the law with impunity or evade prosecution 1. a lawyer may, however, explain the consequences of fraudulent or illegal conduct and may assist the client in making a

good faith effort to determine the validity, scope, or meaning of the law (i.e., looking for a valid loophole) Example: X asks L for tax advice. X found money in the street and wants to know whether it is income for tax purposes. L tells X that it is income. X then informs L that he (X) is not going to report the found money as income and asks L for advice on how to prevent the IRS from discovering the money. It would be inappropriate for L to give such advice and L must so inform X. a. if a client asks for advice on breaking the law or committing fraud, the lawyer is required to explain to the client the relevant limits on the lawyers conduct (e.g., I cannot give you advice on how to evade taxation) 2. if the client insists upon pursuing illegal or fraudulent conduct (or has used the lawyers advise to commit such acts), the lawyer may withdraw and, in some cases, may reveal the crime or fraud (See Rule 1.6(b)(2) and (3)). Example: Same as prior example, but X insists upon evading the tax laws and asks L to continue to represent him. L should withdraw, but is not required to reveal the fraud to the IRS. If L had prepared and filed a tax return for X (before learning of the fraud), L could notify the IRS that he was withdrawing and disavow the return. E. A lawyer must diligently represent a client 1. the lawyer should pursue a clients matter diligently, despite the lawyers own schedule and any normal obstacles Example: L repeatedly fails to meet discover deadlines for no legitimate reason. Although her clients are not injured by such actions, L would be subject to discipline. 2. a lawyer, however, may agree to ordinary continuances, etc., where the client will not be prejudiced (even if the client generally opposes delays) MPRE Question 46 3. a lawyer has a duty to inform (preferably in writing) a client that the lawyer will not handle a matter for the client or is no longer representing the client. Any reasonable doubts will be resolved in favor of the client (e.g., a client may assume that his or her trial

attorney will handle the appeal, unless the lawyer makes clear otherwise). Example: A visits Ls office and asks L to represent him in an automobile accident case. L only handles tax cases, so L declines to represent A. L should (1) advise A to visit other lawyers; (2) advise A of any impending deadlines (e.g., statute of limitations -if the deadline is about to expire, L may be required to do more, such as filing a complaint for A before declining representation); (3) protect As confidences; and (4) notify A in writing that he is declining the representation. 4. Solo practitioners should make arrangements for another lawyer to review his or her files and contact his or her clients in the event the solo practitioner dies unexpectedly. F. A lawyer must keep the client informed 1. this is an affirmative duty (i.e., the lawyer must provide information to the client without request); it requires the lawyer to promptly respond to client requests for information; promptly inform the client of any decision requiring the clients consent; keep the client reasonably informed about the status of the matter; and consult with the client about any limitations on the lawyers conduct mandated by the Rules. Example: L represents A in the case of A v. B. Bs lawyer serves discovery requests on L. L must notify A of the receipt of these requests and send copies to A. 2. a lawyer must promptly inform a client of all offers to plea bargain, all settlement offers (unless the client has previously made clear that such offer would be unacceptable), and the receipt by the lawyer of any client funds (e.g., settlement checks) or property 3. in extraordinary circumstances, a lawyer may delay communication if the lawyer believes the client may react imprudently (e.g., if a mental health worker has advised the attorney that the client may become violent or suicidal upon learning the information) a. a lawyer must withhold information from a client if so ordered by the court (i.e., an attorneys eyes only protective order) G. A lawyers fees and expenses must not be unreasonable

1. a reasonable fee may be based on numerous factors, including the time and labor required, the novelty and difficulty of the issues, the skill required, other work that will have to be turned away (assuming the client knows of this fact), fees customarily charged in the area for such work, the amount involved in the matter, the results obtained, time constraints imposed by the client or circumstances, nature and length of the relationship, the lawyers experience, ability, and reputation, and whether the fee is fixed or contingent. A lawyer may be paid in cash, checks, credit cards, and property. Some payments in property, however, may be considered business transactions between the lawyer and client and, as such, would be governed by Rule 1.8 (requiring fairness, full disclosure, and written consent). Example: A needs a lawyer to handle a difficult legal dispute. A wants to find a lawyer who will devote every minute of the next three months to the dispute. A asks L to take the case. L informs A that he will be forced to turn away $10,000 per month of other work to take the case under such circumstances. A agrees to such terms. L can properly charge A $30,000 for the next three months (and require A to pay a $30,000 retainer); if the case is resolved prior to that time (i.e., it settles), L must refund any unearned fees. 2. except for regular clients that are being represented at their regular rate, a lawyer must communicate to the client the rate and basis of the fee, the scope of the representation, and expenses for which the client will be responsible at the outset of representation (or as soon thereafter as practical); a lawyer must also promptly inform the client of any changes to fees or expenses. a. preferably, this should be in writing; for contingent fees, it must be in a writing signed by the client and must explain how the fee will be calculated (the percentages charged, whether the percentages are calculated before or after expenses), and must notify the client of any expenses the client will be liable for whether or not the client prevails; upon conclusion of a contingent fee case, the lawyer must provide the client with a written account b. if an attorney agrees -- either in an advertisement or in an initial agreement with a client -- to perform a service for a certain flat fee or for a certain hourly rate, the attorney

must comply with such agreement (unless the client expressly agrees to a modification) Example: The law firm of A & B agreed to represent Client in various business matters. The written retainer agreement called for Client to pay A & Bs hourly rates of $180 per hour for a partners time and $110 per hour for an associates time. The representation proceeded. A & B submitted monthly bills, which Client paid promptly. After two years, A & B decided to increase their hourly rates by $10. A & B thereafter billed Client at their new rates, but did not specifically inform Client of the increase. Client continued to pay the bills promptly. Are A & B (as individuals) subject to discipline? Yes, because Client did not consent to the increase. Example: In a proper newspaper advertisement, Attorney states that she will handle uncontested divorces for $400. Client asks Attorney to handle her uncontested divorce for the $400 fee. Attorney accepts the case. Although uncontested, it turns out that Clients divorce involved some complex tax issues for which Attorney was required to conduct 10 hours of research. Attorney bills Client $2,000 for the representation, which is based on Attorneys regular hourly rate. Is Attorney subject to discipline? Yes. 3. contingent fees (cash or property) are proper, except for: a. domestic relation cases (a lawyer may charge a contingent fee if the lawyer is hired simply to collect post-divorce balances due for support, alimony, or marital property settlement under a prior divorce decree) b. criminal cases c. when a contingent fee arrangement is not in the best interests of the client (e.g., client asks lawyer to handle a simple closing, which will take 2 hours, but lawyer insists upon a 33% contingent fee); in such cases, the lawyer must offer the client alternative fee arrangements (e.g., hourly rates or flat fees) Note: there are no numerical caps on contingent fees in the Rules, as long as overall fee is reasonable; some states impose such caps by statute

10

4. a lawyer may split fees with other lawyers in his or her firm without notice to or consent of the client as long as the overall fee is reasonable a. a lawyer may not split fees with a non-lawyer witness or consultant under any circumstances 5. a lawyer may split fees with lawyers outside of his or her firm if the overall fee is reasonable and: a. the division is in proportion to the services performed (i.e., two lawyers truly sharing the work) or each lawyer assumes responsibility for the representation, and b. the client agrees to the arrangement, including the share each lawyer will receive, and the agreement is confirmed in writing. Confirmed in Writing. Whenever this phrase is used in the Rules, it means a writing prepared by the client or a written confirmation of an oral agreement prepared by lawyer and promptly transmitted to the client. Example: L is handling a complex domestic and international tax dispute for B. Although L is experienced in domestic tax matters, he is not an expert in international tax matters. L would like to ask A, an international tax lawyer in another firm, to assist L in the matter. L will handle the domestic tax issues and A will handle the international tax issues. They will split the fee in proportion to the work each performs. This is proper if L follows the disclosure and consent rules stated above. MPRE Question 23 Example: B asks L to represent him in a negligence case. L is an estate planning lawyer and does not handle negligence cases. L wants to refer the case to A, Ls law school classmate. As a condition of such referral, L wants A to agree to give L 10% of any recovery (i.e., a true referral fee). This is proper only if the lawyers follow the disclosure and consent rules stated above. c. a lawyer may enter into a non-exclusive reciprocal referral agreement with another lawyer or a nonlawyer professional; clients obtained through such agreements must be informed of the nature of such agreement

11

H.

A lawyer must protect client confidences 1. There are two rules dealing with client confidences: Rule 1.6 and the attorney-client privilege: Rule 1.6 Confidences

Attorney-Client Privilege Rule 1.6 Confidences

a.

Rule 1.6: This rule precludes an attorney from voluntarily disclosing information or using information (to the clients detriment) relating to the representation of the client (regardless of its source); Exceptions: The lawyer may reveal such information if: i. the client gives informed consent to such disclosure ii. the disclosure is impliedly authorized in order to carry out the representation (e.g., the lawyer admits a fact in open court that is beyond dispute or the lawyer makes a statement of fact in settlement negotiations) iii. to prevent reasonably certain death or substantial bodily harm (note: there is no longer a requirement that there be a crime or that the death or injury be imminent) iv. to prevent the client from committing a crime or fraud that is reasonably certain to result in substantial injury to the financial interests or property of another and in furtherance of which the client has used or is using the lawyer's services (this has been the ABA Model Rule position since August 2003 and is the majority view)

b.

12

v.

to prevent, mitigate or rectify substantial injury to the financial interests or property of another that is reasonably certain to result or has resulted from the client's commission of a crime or fraud in furtherance of which the client has used the lawyer's services (this has been the ABA Model Rule position since August 2003 and is the majority view) to secure legal advice about the lawyer's compliance with these Rules (lawyer may disclose only to the extent necessary to obtain advice) to establish a claim or defense on behalf of the lawyer in a controversy between the lawyer and the client, to establish a defense to a criminal charge or civil claim against the lawyer based upon conduct in which the client was involved, or to respond to allegations (even before a suit is filed) in any proceeding concerning the lawyer's representation of the client (lawyer may disclose only to the extent necessary to prosecute or defend); examples: a. b. client sues lawyer for malpractice lawyer sues client for fees c. third party lawyer for committing fraud with client sues

vi.

vii.

MPRE Question 15 d. disciplinary proceeding against lawyer e. viii. lawyer charged with crime arising out of representation

to comply with other law or a court order.

13

Notes: a lawyer is never obligated to reveal Rule 1.6 information under the Rules (except to remedy perjury or fraud on the court); Rule 1.6 continues after the representation terminates and after the client dies. c. The attorney-client privilege: this evidentiary rule prevents compelled disclosure (e.g., discovery, police interrogation, trial testimony, grand jury testimony) by an attorney of confidential communications with a client for the purpose of seeking legal advice i. the attorney-client privilege applies only to communications between a client (and his or her representatives) and a lawyer (and his or her representatives); it applies only when legal advice (as opposed to business or personal advice) is sought ii. the attorney-client privilege does not apply to information obtained from third parties or from public records Exceptions to the attorney-client privilege: a. if the client seeks legal advice related to an ongoing crime or fraud b. there is no privilege among joint clients (but the privilege remains as to outsiders) c. the client waives the privilege d. the communications were not confidential when made (discussions took place in a crowded store) or the client has since made them public d. Differences between Rule 1.6 and attorney-client privilege: i. All attorney-client information is protected by Rule 1.6, but not all Rule 1.6 information is protected by the attorney-client privilege; in other words, the attorney-client privilege is a small subset of Rule 1.6

iii.

14

ii. The attorney-client privilege applies only to information the attorney gets from the client (or the clients representatives) iii. The attorney-client privilege protects against compelled disclosure; Rule 1.6 prohibits voluntary disclosure by the lawyer iv. Rule 1.6 is not a proper objection at trial or in discovery, but the attorney-client privilege is a proper objection v. Both rules continue after the client dies (although there is an exception to the attorney-client privilege in cases of testamentary dispositions) or the representation terminates

I.

A lawyer who represents an entity (e.g., a corporation) represents the entity and not its officers, directors, employees, etc. 1. if a lawyer learns that a corporate employee has breached his or her obligations to the corporation or violated the law in such a fashion that the act may be imputed to the corporation and is likely to result in substantial injury to the corporation, the lawyer should proceed as follows (minimizing disruption to the entity as much as possible): a. b. ask the employee to reconsider (if possible); if that fails, advise the employee to get a separate legal opinion (assuming there is a dispute as to the legality of the act); c. if that fails (and the matter is sufficiently serious), disclose the matter to the employees boss (all the way to the CEO, if necessary); the Rules give lawyers some discretion to take this step; for securities lawyers, the Sarbanes-Oxley Act requires lawyers to refer such matters to the Chief Legal Officer; d. if that fails, disclose the matter to the Board of Directors; the Rules give lawyers some discretion to take this step; for securities lawyers, the Sarbanes-Oxley Act requires lawyers to refer such matters to the Board, an audit committee, or a committee of outside directors;

15

e. if that fails and the entity is clearly violating the law, the lawyer may (but is not required to) reveal information relating to the representation to the appropriate authorities or injured parties, but only if and to the extent the lawyer reasonably believes necessary to prevent substantial injury to the organization; for securities lawyers, the Sarbanes-Oxley Act gives lawyers the discretion (but does not require them) to disclose such matters to the SEC. Example: L represents ABC Corp. X is vice-president of ABC. While working with X on the sale of ABC securities, L learns that X is misrepresenting the value of the securities to the public in violation of federal securities law. Such illegal conduct could be imputed to ABC and is quite serious. L informs X of the violation, but X ignores L. What should L do? (1) explain to X that L represents ABC and not X; (2) advise X to reconsider or seek a second legal opinion; (3) if that fails, reveal the fraud to Xs superior (the Sarbanes-Oxley Act requires this act), and continue all the way to the Board, if necessary to resolve the matter; (4) if that fails, L may (but is not required to) reveal the fraud to the SEC (or other appropriate party) if L reasonably believes necessary to prevent substantial injury to the organization. f. a lawyer may not reveal information outside the organization relating to the lawyer's representation of an organization to investigate an alleged violation of law, or to defend the organization or an officer, employee or other constituent associated with the organization against a claim arising out of an alleged violation of law Example: L is hired to represent ABC Corp. in a securities fraud case asserted by the SEC. During Ls investigation, she discovers that ABC violated federal securities law. L is prohibited from disclosing this fact outside the corporation. g. a lawyer who reasonably believes that she has been discharged because of the lawyer's actions taken pursuant to Rule 1.13 (as stated in the preceding paragraphs of this Section), or who withdraws under circumstances that require or permit the lawyer to take action under that Rule, shall proceed as the lawyer reasonably believes necessary to assure that the organization's highest authority is informed of the lawyer's discharge or withdrawal.

16

Example: L represents ABC Corp. in a real estate transaction. During the transaction, L learns that X, vicepresident of ABC, is attempting to defraud the other party in the transaction. L confronts X with this fact. As a result, X fires L. L should disclose these facts to ABCs president or board of directors. 2. when a corporations lawyer deals with corporate officers, directors, employees, or agents, the lawyer shall inform such individuals that the lawyer represents the entity, and not them as individuals, where the individuals interests are adverse to the entity; the lawyer may advise the individual to obtain independent counsel (but should provide no other advise) 3. a lawyer may jointly represent a corporation and one or more of its officers, directors, employees, etc., if such representation can be made without violating the rules regarding conflicts of interest between two current clients (infra) 4. for purposes of the attorney-client privilege, communications between a corporations lawyer and corporate officers, directors, and employees are protected from disclosure if the communications relate to issues within the scope of the employees work and occur while the employee is employed by the corporation

J.

A lawyer must avoid conflicts of interest (which subject lawyers to discipline, disqualification, and possibly civil liability for breach of fiduciary duty) 1. Conflict Between Two Clients of the Same Firm a. Conflict Between Two Current (or Regular) Clients i. Rule: as part of the lawyers duty of loyalty, a lawyer shall not represent a person with an interest directly adverse (e.g., opposite side of transaction or litigation) to that of another current (or regular) client, unless both clients give informed consent in writing and the lawyer reasonably believes that the lawyer will be able to provide competent and diligent representation to both clients (i.e., would a disinterested lawyer advise against the representation?). The lawyer may not represent two concurrent clients if such joint representation is prohibited by other law (i.e., some states prohibit joint representation in certain criminal matters). In addition, the lawyer may not

17

represent both clients if one of them is asserting a claim against the other in litigation. If a lawyer is disqualified under this rule, all lawyers in the lawyers firm are disqualified. Example: L represents A in a pending automobile accident lawsuit against X. B has made an offer to buy As house. B asks L to represent B in the closing. L may NOT represent B in the closing, unless both clients consent in writing and neither client will be adversely affected (which may well be true if both give informed consent). Example: L represents A in a pending automobile accident lawsuit against X. X asks L to represent X in the same lawsuit. L may NOT represent X, even if both clients consent. Tip: a lawyer may not sue a current client, even if the suits are unrelated. Example: L represents A in a pending automobile accident lawsuit against X. While As case is pending, W asks L to represent W in an unrelated real estate lawsuit against A. L may NOT represent W, even if both clients consent. MPRE Question 50 Rule: a lawyer may usually represent two parties (in different lawsuits) that are asserting opposite positions on legal issues; it may be inappropriate to do so, however, where there is a significant risk that the lawyers action on behalf of one client may materially limit the lawyers effectiveness in representing another client. Example: L represents A in a securities fraud appeal pending in the Fourth Circuit. The statute of limitations for securities fraud is uncertain; in As appeal, L argues that it is 3 years. L also represents B in a similar appeal in the Fourth Circuit. In Bs appeal, L argues that the statute of limitations is 5 years. This may be inappropriate (because the decision in one case may be binding or highly persuasive on the other case), but probably would be proper if the cases were pending in different

18

appellate courts (because the authority in one case would not be binding in the other). Imputed Disqualification: If a lawyer is disqualified under this rule, all lawyers in the lawyers firm are disqualified. b. Conflict Between a Current Client and a Former Client (no current or ongoing representation) i. a lawyer may represent a person in litigation or a transaction against a former client as long as the matter is not substantially related to the matter the lawyer handled for the former client; if the matters are substantially related, the lawyer must get the informed written consent of the former client. Example: L represented A, a physician, in an automobile accident lawsuit against X. The lawsuit concluded five years ago, and L has not represented A since that time. B asks L to represent B in a medical malpractice case against A. L may represent B, since the two matters are not substantially related. Example: L represented ABC Corp. in its negotiation of an employment contract with X. The contract negotiations were successful and concluded 5 years ago. L has not represented ABC Corp. since that time. X wishes to sue ABC Corp. for breach of his employment contract (the one drafted by L). X asks L to represent X in a suit against ABC Corp. L may NOT represent X, because the two matters are substantially related, unless ABC consents in writing. MPRE Questions 17 & 18 Rule: a lawyer who has confidential information about a former client may not use that information to the disadvantage of that former client, unless permitted to do so by Rule 1.6 or if the information has become public Example: Five years ago, L prepared a will for A. During this process, L learned a great deal about As secret assets. X now asks L to sue A to collect a promissory note. L may not represent X because L

19

has confidential information about A, Ls former client, that L could use to the disadvantage of A. Imputed Disqualification: If a lawyer is disqualified under this rule, all lawyers in the lawyers firm are disqualified. c. Lawyers are required to have procedures in place to detect conflicts of interest before such conflicts arise; failure to have such a system in place may, alone, subject a lawyer to discipline d. Conflict Between a Current Client and a Third Party i. a lawyer shall not represent a person if there is a serious risk that the representation will be materially limited by the lawyers responsibilities to another client, a former client, a third person, or the lawyers own interests, unless the client gives informed consent in writing and the lawyer reasonably believes that the client will not be adversely affected. Example: A asks L to represent him in a quiet title lawsuit against X. The property is Blackacre. A claims ownership, as does X. X is Ls father-in-law. L should decline to represent A because his representation of A would be materially limited by his relationship to X. This is probably true even if A gives informed consent. Example: A asks L to represent him in a quiet title lawsuit against ABC Corp. The property is Blackacre. A claims ownership, as does ABC. L owns 10% of ABCs stock. L should decline to represent A because his representation of A would be materially limited by Ls own interests. This is probably true even if A gives informed consent. Example: A and B are being tried jointly for burglary. A and B ask L to represent them jointly. L should decline, unless (1) there are no existing or potential conflicts between A and B (i.e., A is prepared to plea and testify against B); (2) both parties consent knowing the implications (i.e., L will be forced to withdraw if a conflict between A and B ensues) and risks (i.e., there is no attorney-client privilege between joint clients) of joint

20

representation; and (3) the lawyer reasonably believes that neither client will be adversely affected. Note that it is rarely appropriate for a lawyer to represent joint clients in a criminal matter; lawyers have more flexibility to represent joint clients in civil cases. Example: Lawyer A represents C in litigation against D. D asks Lawyer B, who is Lawyer As mother, to represent D. Lawyer B may not represent D unless a reasonable lawyer would conclude that such representation is feasible and D gives informed written consent to the conflict. However, other lawyers in Bs firm may represent D, as such conflicts are personal. e. Conflicts Between the Lawyer and Client i. Business Transactions Between Lawyer and Client: a lawyer shall not enter into a business transaction (other than transactions in the ordinary course of business -- bank accounts at clients bank) or acquire a property interest adverse to a client, unless: a. the terms are fair and fully disclosed in writing in a manner that can be reasonably understood by the client b. the client is advised in writing of the desirability of seeking and is given a reasonable opportunity to seek the advice of independent legal counsel, and c. the client informed consent in a signed writing gives

ii. Using Client Information: a lawyer may not, without the informed consent of the client, enter into transactions disadvantageous to the client Example: L learns that her client, A, is interested in buying Blackacre. L buys Blackacre and sells it to A at a higher price. L is subject to discipline and civil liability for breach of fiduciary duty (L would be required to disgorge profits to A)

21

Example: L learns that her client, A, is interested in buying Blackacre. L buys Whiteacre, an adjoining parcel, speculating that As purchase of Blackacre will increase the value of Whiteacre. A had no interest in acquiring Whiteacre. Ls speculation paid off, as L sells Whiteacre for a profit. L is probably NOT subject to discipline (because A was not disadvantaged), but would be subject to civil liability for breach of fiduciary duty (L would be required to disgorge profits to A). iii. a lawyer shall not solicit any substantial gift from a client or prepare an instrument (e.g., will, trust, deed) giving the lawyer (or a close family member) a substantial inter vivos or testamentary gift, except: a. where the donor is a close family member of the lawyer (i.e., a lawyer may draft a will for his parents in which he receives a bequest) b. a lawyer may accept simple, unsolicited gifts (e.g., birthday, holiday) from clients that do not require the preparation of an instrument; a lawyer may even accept large, unsolicited gifts (e.g., a new car for outstanding results in a lawsuit) from a client, but the client should seek the advise of an independent lawyer prior to making such gift (although the donee lawyer is not required to inform the client of this fact) iv. a lawyer shall NOT make or negotiate an agreement for the literary or media rights arising out of the representation of a client a. a client cannot consent to this conflict b. after termination of the representation, the lawyer may acquire such rights (this assumes that the person is no longer a client; hence, this is nothing but a business transaction between a lawyer and a non-client)

22

Example: L represents Charles Manson in his murder trial. Manson has no money to pay L, so Manson agrees to assign the rights to a book and movie about his crimes and trial to L. L is subject to discipline. Manson cannot consent to this conflict. Once Ls representation of Manson concludes, however, L may acquire such rights. Example: A is an aspiring author. A has written a screenplay and hopes to sell it to a television network. A hires L to negotiate a contract with NBC. A has no money, so A offers to give L 10% of the profits from the movie for Ls fee. Assuming 10% is reasonable, Ls actions are proper. This is nothing but a contingent fee in property (as opposed to cash), and is not considered the acquisition of literary rights for purposes of the conflict of interest rules MPRE Question 10 v. a lawyer shall not provide financial assistance to a client in connection with litigation, except: a. the lawyer may advance the court costs and expenses of litigation, contingent upon outcome of the matter (but not other costs, such as the clients rent payments, food, medical bills, etc.) b. the lawyer may pay the court costs and expenses of litigation for an indigent client (but not other costs, such as the clients rent payments, food, medical bills, etc.) Example: Lawyer represents A, an indigent, in a dispute with her landlord. Lawyer pays As court costs and gives A $100 to buy food, because A has no money. L is subject to discipline for giving A the $100. c. for non-litigation clients, a lawyer may make loans, etc. to such clients if the lawyer complies with the rules for entering into business transactions with a client

23

vi.

a lawyer shall not accept compensation for representing a client from someone other than the client (e.g., an insurer, parent, employer), unless: a. the client gives informed consent in writing b. the third party does not interfere with the relationship or the lawyers independent judgment c. the clients Rule 1.6 information is protected (i.e., the third party is not entitled to such information)

Example: Dad hires (and pays) L to represent his adult son, S, in Ss trial for drunk driving. Dad tells L that he (Dad) will call the shots and expects to be kept apprised of all information. L cannot represent S under these circumstances. vii. a lawyer shall not represent joint clients in aggregate settlements or plea bargains unless each client gives informed consent in a writing signed by the client. The lawyer must provide each client with all information about the settlement, including what each client is receiving Example: L represents A, B, and C in a negligence case. L reaches an aggregate settlement with the defendant. L has split the settlement into three unequal portions, and L intends to tell each client only about the portion he or she is to receive, but not the portions the other clients will receive. L is concerned that if A learns that B and C received larger portions, A will reject the deal. L is subject to discipline because the clients are not fully informed of all the details of the settlement. viii. Rule: a lawyer shall not make a prospective agreement limiting the lawyers liability for malpractice, unless the client is independently represented Example: Ls standard retainer agreement contains a clause in which clients agree to release L from all ordinary negligence. L is subject to discipline.

24

Rule: a lawyer may agree with a prospective client to arbitrate malpractice claims. Rule: a lawyer may not settle a malpractice claim with a client without first advising the client in writing to seek independent legal advice MPRE Question 45 ix. a lawyer shall not acquire a proprietary interest in the subject matter of a clients lawsuit (e.g., collection company sells or assigns its claims to L for 50 cents on the dollar), except: a. a lawyer may acquire an attorneys lien (where authorized by state law) to secure the lawyers fee a lawyer may contract with a client for a contingent fee in cash or property Example: L agrees to represent A in As negligence case against D. L and A agree to a 25% contingent fee. Assuming the fee is reasonable, Ls actions are proper. Example: L agrees to represent A in As quiet title action against D. The case involves the ownership of Blackacre. L and A agree that, if A prevails, L will receive 25% of Blackacre. Assuming the fee is reasonable, Ls actions are proper. c. L may take a security interest or mortgage in property that is not the subject of the litigation to secure payment of fees. Example: L agrees to represent A in As contract case against D. A agrees to pay L $200 per hour. To secure payment, A agrees to give L a mortgage on As home (which is not at issue in the litigation). Ls actions are proper. x. A lawyer may not have sexual relations with a client unless such relationship

b.

25

existed prior to the representation. This conflict is NOT imputed to other members of the lawyers firm. f. Conflicts When Lawyers Change Firms i. Conflicts that Follow a Lawyer to his or her New Firm (the Typhoid Mary problem) Hypo: L is an associate at the Smith Law Firm. The Smith Firm represents P in a contract action against D. While the case is pending, L leaves the Smith Law Firm and joins the Jones Law Firm. The Jones Law Firm represents D in the contract action. (1) Can L work on the case at the Jones Law Firm? (2) Can the Jones Law Firm continue to represent D? (1) because this is the same (or substantially related) matter that Ls former firm handled, and because Ds interest is materially adverse to Ps interest, L may not represent D if L acquired material Rule 1.6 information about P while at the Smith Law Firm. (2) if L is barred from representing D, so is the entire Jones Law Firm (i.e., this conflict is imputed to the new firm; hence, Typhoid Mary)

ii. a lawyer who has Rule 1.6 information about a former client may not use this information in a manner disadvantageous to the client (unless the information has become public) or reveal Rule 1.6 information iii. Conflicts at the Former Firm after the Lawyer Leaves (the Lingering Conflict problem). Hypo: L is a partner in the Smith Law Firm. While at the Smith Law Firm, L represents P in a contract action against D. L leaves the Smith Law Firm and joins the Jones Law Firm. L takes P (and Ps case against D) with him to the Jones Law Firm. The Smith Law Firm no longer represents P in any matter. (1) May the Smith Law Firm now represent D in her action against P? (2) May the Smith Law Firm represent X in an unrelated case against P?

26

(1) when a lawyer leaves a firm (and takes a client with her), the firm is not prohibited from representing a person with an interest materially adverse to that former client (if its a present client, Rule 1.7(a) applies), unless the representation involves the same or a substantially related matter and any lawyer remaining in the firm has Rule 1.6 information about the former client (this conflict may be waived by the former client). Thus, in the hypo, the Smith Law Firm could not represent D in P v. D (which is the same case that L handled for P at the Smith Law Firm) if any lawyer at the Smith Law Firm has Rule 1.6 information about P (which is quite likely). (2) as to the unrelated case, the Smith Law Firm could represent X in his suit against P. iv. Imputed Disqualification a. if a lawyer is disqualified because of a conflict of interest, the lawyers entire firm is disqualified, except for Rule 1.8(j) conflicts (sexual relationships with clients), and Rule 1.11 or 1.12 conflicts (conflicts caused by lawyers moving between the government and private practice). A conflict will not be imputed to the firm, however, if it is based on a personal interest (e.g., the political beliefs) of the prohibited lawyer and does not present a significant risk of materially limiting the representation of the client by the remaining lawyers in the firm. Moreover, non-lawyers (e.g., secretaries, clerks, etc.) that have conflicts may be screened. b. Firm means law firms, corporate legal departments, legal services organizations, etc. g. Conflicts When Lawyers Move Between the Government and Private Practice i. conflicts when a lawyer moves from the government to private practice: a lawyer who leaves government employment (as a lawyer or

27

otherwise) shall not represent a private client in connection with a matter on which the lawyer participated personally and substantially at the government, unless the government agency gives informed consent Example: L works as a lawyer for the IRS. While with the government, L reviews Xs tax file for purposes of preparing a Private Letter Ruling. The PLR denied Xs request for a tax deduction. L leaves the government and joins the Smith Law Firm. X asks L to represent him in court in an action against the IRS related to the same deduction issue. L is prohibited from representing X (unless the IRS consents) because L personally and substantially participated in Xs matter as a government employee. Example: Same facts as prior example, but Y asks L to represent Y in a lawsuit against the IRS. Although Ys dispute with the IRS existed at the time L was employed by the government, L played no role in the matter and has no confidential government information about Y. L may represent Y. Imputed Disqualification: even if the former government lawyer is disqualified, this conflict is not imputed to the lawyers firm, if the firm screens (a Chinese Wall) the lawyer: a. from any participation in the matter b. from sharing any part of the fee from the representation (although a partner can continue to share in overall partnership profits from an existing partnership agreement, which may include such fee) c. prompt written notice is given to the agency.

ii. conflicts when a lawyer moves from the government to private practice: a former government lawyer who has confidential government information (i.e., information the government is prohibited by law from disclosing or has a legal privilege not to disclose to the public) about a person may not represent a private client

28

whose interests are adverse to that person in a matter in which the confidential government information could be used to the disadvantage of that person Example: L works for the IRS. While employed by the government, L works on Xs tax file. During this work, L learns that X makes most of his money selling fraudulent securities. This is confidential IRS information (i.e., the IRS is prohibited by law from disclosing this information to the public). L leaves the IRS and joins the Smith Law Firm. Y asks L to represent Y in a fraud suit against X. L cannot undertake such representation if she has confidential government information (which appears so) about X and this information could be used to the disadvantage of X in Ys suit against X Imputed Disqualification: The remaining lawyers at the Smith Law Firm, however, could represent Y in the case of X v. Y, if L is screened from participation in the case and is apportioned no part of the fee therefrom. iii. conflicts when a lawyer moves from private practice to the government: a lawyer who participated personally and substantially in a matter while in private practice shall not work on the matter as a government official (lawyer or otherwise), unless, under law, the lawyer is the only official authorized to so act (this exception is quite rare) iv. government lawyers (including judges, arbitrators, mediators and third-party neutrals) negotiating private employment: a government lawyer (including a judge, arbitrator, mediator, or third-party neutral) shall not negotiate private employment with any person involved as a party or lawyer in a matter in which the government lawyer (including a judge, arbitrator, mediator or third-party neutral) is participating personally and substantially Example: L is an attorney for the IRS. While handling a case against XYZ Corp., L is asked to join XYZs legal department. L begins negotiating for the job while the case is pending. This is improper.

29

Example: J is a judge presiding over the case of X v. Y. Y is represented by the Smith Law Firm. J cannot negotiate private employment with X, Y, or the Smith Law Firm while the matter is pending before J. v. law clerks negotiating private employment: a judges law clerk may negotiate with any party or firm if he or she so notifies the judge Example: L is a clerk for Judge J. Judge J is residing over a major trial in which X is the defendant. X is represented by the Smith Law Firm. L has been actively involved in the case as a clerk. L may negotiate with the Smith Law Firm for private employment if L notifies J of this fact vi. judicial officials, mediators, and third-party neutrals representing private clients: a judge, mediator, or third-party neutral shall not represent a client on a matter in which he or she participated personally and substantially as a judge, arbitrator, magistrate, law clerk, mediator, thirdparty neutral, etc., unless all of the parties to the matter give informed consent confirmed in writing a. a judge does not participate personally and substantially in a matter simply because she holds an administrative position over the entire court (e.g., chief judge) or because she is one member of a court in which the action is pending (e.g., one of 11 judges on the Fourth Circuit). In other words, the judge is generally disqualified only if she presided over the case MPRE Question 6 Imputed Disqualification: The remaining members of the judges (or mediators or third-party neutrals) law firm are not disqualified, if that person is screened from any participation in the matter, is apportioned no part of the fee, and written notice is given to the parties and the appropriate tribunal. vii. a party-arbitrator (chosen by a party as its arbitrator) may represent that party in later proceedings in the same matter

30

K.

A lawyer must safeguard a clients property and funds (the Client Trust Fund Account) 1. the rule with regard to client trust fund accounts is zero tolerance; there must be no commingling of funds (client and non-client); the lawyer cannot move money out of such accounts until earned or the client approves; the lawyer must keep complete, accurate and up-to-date records of client trust fund accounts (and maintain them for 5 years); even bookkeeping neglect or sloppiness will subject the lawyer to discipline (despite the fact that the client was not injured) a. A lawyer may deposit personal funds into a client trust fund account to pay bank service charges. b. A lawyer shall deposit retainer advances in the client trust fund account and must withdraw those funds as earned or expended. 2. a lawyer must promptly inform the client of the lawyers receipt of any property belonging to the client; the lawyer must then promptly deliver the property to the client (or as the client instructs) and must provide the client with a full accounting, where requested a. Example: L receives a settlement check. L must deposit the check in the clients trust fund account and immediately notify the client that it has arrived.

MPRE Question 26 b. for tangible property, earmark the property and keep it safe the lawyer must

Example: L receives a diamond ring belonging to the client at 6:00 p.m. on Friday. L attempts to notify the client of receipt, but is unable to reach her. L must segregate the ring from other property, earmark it (put it in an envelope with the clients name), and keep it safe (put it in the office safe or a safe deposit box), and then notify the client first thing Monday morning. c. if a dispute arises between the attorney and client over ownership of funds, the lawyer should deliver to the client any funds which the parties agree belong to the client, but the lawyer should retain the disputed portion in the client trust fund account

31

Example: L agrees to represent A in her personal injury suit against B for a 25% contingent fee. A receives a judgment of $100,000. A believes that this award would have been much larger had L been better prepared for trial. Thus, A tells L: 25% is way too much; I wont pay you a dime more than 10%. L receives the $100,000 check the next day and immediately notifies A of its receipt. What should L do? (1) deposit the check in Ls trust fund account; (2) immediately send A $75,000 (the undisputed portion belonging to A); (3) transfer $10,000 (the undisputed portion belonging to L) from the trust fund account to the lawyers office account; (4) as for the remaining $15,000, L has two choices: transfer it to A or hold it in the client trust fund account until the dispute is resolved. MPRE Question 5 3. retainers a. non-refundable retainers are generally impermissible: A retains L to represent A in a contract dispute. Because A is a new client, L insists upon a retainer of $5,000 to handle the matter. L deposits the money in the client trust fund account. The matter settles quickly, at a time when L had incurred only $2,000 in fees and expenses. L must transfer the remaining $3,000 to A b. a lawyer must transfer retainer funds from the client trust fund account to the office account as they are earned: A retains L to represent A in a contract dispute. Because A is a new client, L insists upon a retainer of $5,000 to handle the matter. L deposits the money in the client trust fund account. During the first month of the representation, L incurs $750 in fees and $100 in expenses. L must transfer $850 from the client trust fund to Ls office account; failure to do so constitutes commingling, and subjects L to discipline, even though A was not injured.

4.

third-party claims to client funds: where a third party has a nonfrivolous claim to client funds held by the lawyer, the lawyer must refuse to surrender the property to the client until such claims are resolved. In some cases, this may require the lawyer to seek action in court (e.g., interpleader) for a resolution.

L.

A lawyers duty or option to withdraw from representing a client

32

a. Mandatory Withdrawal: A lawyer must withdraw (or must not accept representation) if: 1. the representation will result in a violation of the Rules or Law by the lawyer Example: Client insists that Lawyer file suit against X. Lawyer believes that Clients suit is frivolous and is being asserted solely to harass X. Lawyer must withdraw. 2. the lawyers physical or mental condition materially impairs the lawyers ability to represent the client 3. the lawyer is discharged (a lawyer may always be fired, even if working on a contingent fee basis); the lawyer, however, is entitled to fees earned up to the time of discharge (this would be quantum meruit recovery in a successful contingent fee case)

b.

Permissive Withdrawal: A lawyer may withdraw if: 1. the client persists on a course of action the lawyer reasonably believes is criminal or fraudulent (if the representation will cause the lawyer to violate the Rules or the law, the lawyer must withdraw) the client has used the lawyer to commit a crime or fraud the client insists on taking action the lawyer considers repugnant or with which the lawyer has a fundamental disagreement the client breaches the lawyer-client agreement (e.g., the client fails to pay the lawyer); in such cases, the lawyer must give the client warnings before withdrawing the representation will result in an unreasonable financial burden to the lawyer (particularly if the matter has been rendered unreasonably difficult by the client) or the clients conduct is making it unreasonably difficult for the lawyer to represent the client effectively and competently Example: A has been representing C in litigation. During protracted pretrial proceedings, C complained bitterly about the time and expense involved and insisted that A take steps to terminate pretrial proceedings. A believes that a termination of such proceedings will jeopardize Cs

2. 3.

4.

5.

33

case. A feels that the case cannot be adequately prepared without at least six more months of pretrial proceedings. C tells A to forego all further pretrial proceedings and schedule the case for an immediate trial. Under these circumstances, is it proper for A to withdraw? Yes, because Cs conduct makes it unreasonably difficult for A to represent C effectively and competently. 6. 7. c. 1. 2. the client consents to the lawyers withdrawal other good cause exists Duties Upon Withdrawal: The lawyer shall give reasonable notice of withdrawal to the client The lawyer shall give the client time to find a new lawyer (the lawyer may have to stay on until a new lawyer is hired) The lawyer confidences must protect the clients property and

3. 4.

The lawyer shall promptly surrender the clients files and property to the client or the new lawyer (some states have statutory attorney lien statutes that allow retention of the file) The lawyer shall promptly refund any unearned fees or expenses. d. If litigation has commenced, the lawyer must obtain leave of court to withdraw, regardless of whether such withdrawal is mandatory or permissive. If the court denies such leave, the attorney must continue to represent the client competently

5.

M.

A lawyers duty when buying or selling a legal practice 1. a lawyer may sell his or her law practice, or an area of practice, including good will, if the lawyer ceases to engage in private practice of law in the geographic area (e.g., the lawyer is relocating to another state, retiring, or leaving private practice) or in the area of practice that has been sold a. the selling lawyer must give written notice to his or her clients of the proposed sale, the fact that the

34

client may retain other counsel (and take possession of the clients file), and the fact that the clients consent to the sale (to the new representation) will be presumed unless the client objects within 90 days. b. c. if a client cannot be given actual notice, a court must approve the substitution the purchasing lawyer may make the selling lawyer sign a reasonable covenant not to compete

2. a buying lawyer cannot raise the fees charged by the selling lawyer. Example: L is retiring and offers to sell his practice to A for $100,000. A has no cash to buy the practice, but figures he can borrow the $100,000 from a bank and payoff the loan by increasing the fees charged to Ls former clients. This is improper. N. Duties to a Prospective Client 1. A person who discusses with the lawyer the possibility of forming a lawyer-client relationship is a prospective client, even if no relationship ensues. 2. The lawyer owes prospective clients the same duties of competency, confidentiality, loyalty (i.e., avoiding conflicts of interest), and preservation of property that it owes all clients. 3. A prospective client should be treated as a former client for purposes of conflicts of interest; i.e., a lawyer who obtains confidential information from a prospective client (who does not retain the lawyer) may not represent a different person in the same or a substantially related matter if the confidential information could harm the prospective client, unless both clients give informed written consent. The lawyers firm is also disqualified, unless: a. the lawyer who communicated with the prospective client took care to avoid exposure to any more confidential information than was necessary to determine whether to represent that person; b. c. the lawyer is timely screened from the matter and will not directly share in any fee from the matter; and written notice is given to the prospective client.

35

II.

A Lawyers Duty as Counselor, Evaluator, Mediator, or Third-Party Neutral (Duties to Client and/or Third Parties) A. a lawyer may give nonlegal (e.g., business, political, personal) advice to a client, in addition to legal advice (e.g., a lawyer may advise a client to settle because such a settlement would be good for business or a lawyer may recommend that a client get parental counseling) MPRE Question 7 B. C. a lawyer may advise a client to seek nonlegal help (e.g., a referral to a mental health specialist or social worker) a lawyer may evaluate a matter affecting a client for use by a third party (e.g., the most common evaluation is an opinion letter or title inspection prepared by the lawyer for a third person evaluating the clients lawsuit, title to land, etc.) if: 1. the evaluation is compatible with the lawyers relationship with the client (i.e., a lawyer shall not give evaluations to third parties that contain misrepresentations or material omissions) and 2. when the lawyer knows or reasonably should know that the evaluation is likely to adversely affect the clients interests, the lawyer shall not provide the evaluation unless the client gives informed consent D. a lawyer may serve as a third-party neutral, such as an arbitrator or mediator, to assist two or more persons reach a resolution of a dispute. In such role, the lawyer does not represent any of the parties as clients and should make this clear to everyone, particularly if there is uncertainty about the lawyers role. A lawyer who has served as a thirdparty neutral in a matter may not later represent any of the parties in the matter. Such conflict is imputed to the third-party neutrals firm, unless the third-party neutral is properly screened from the matter.

III.

A Lawyers Duty in Litigation A. A lawyers duty to the tribunal (e.g., the court) 1. a lawyer shall not file frivolous claims or defenses (i.e., those not well-grounded in law or fact or those asserted for delay, harassment, etc.) a. a good faith argument modification of the law is not frivolous for reversal or

36

i.

this probably means subjective good faith only and not probability of success

b. a criminal defense lawyer may require the prosecution to prove all elements of its case even if the defendant has no valid defense c. frivolous pleadings also violate FRCP 11 and thus subject a lawyer and the lawyers firm to litigation sanctions (usually monetary sanctions) 1. Safe-Harbor: a lawyer may avoid sanctions under FRCP 11, if the lawyer within 21 days of being served with a motion for FRCP 11 sanctions, withdraws or amends (so as to comply with FRCP 11) the pleading.

d.

if a lawyer believes from the outset that a clients motive is harassment or that the clients claims are frivolous, the lawyer must decline the representation

2.

a lawyer may delay proceedings only if he or she has a legitimate purpose a. illegitimate purposes: my client will gain financially from a delay, we can out wait our opponent, everyone does it, and the courts dont seem to care b. legitimate reasons: more evidence is needed; an essential witness is out of town 3. a lawyer shall not make false statements of fact or law to the court or fail to correct a false statement of fact or law previously made to the tribunal by the lawyer 4. but otherwise a lawyer is under no duty to disclose facts to a tribunal, except a. the proceeding is ex parte

5. a lawyer must disclose legal authority from a controlling jurisdiction to a tribunal if that authority is directly adverse to the position of the lawyers client and is not disclosed by his or her opponent a. controlling jurisdiction means authority plus some very persuasive authority binding

37

Example: In a federal question case pending in the Western District of Virginia, federal statutes, federal administrative regulations and decisions, U.S. Supreme Court decisions, Fourth Circuit decisions, and W.D. Va. decisions are controlling authority. In a diversity case in the same court in which Virginia law governs, Virginia statutes, Virginia administrative regulations and decisions, Virginia Supreme Court decisions, and Virginia Court of Appeals decisions, and possibly even Virginia Circuit Court decisions are controlling authority. 6. a lawyer must not offer evidence the lawyers knows is false:

MPRE Question 41 a. if the lawyer knows the client intends to commit perjury, the lawyer must: i. attempt to dissuade client from committing perjury ii. if that fails, the lawyer must withdraw (if permitted by the court) or if not permitted, the lawyer must remedy any perjury that occurs b. if the lawyer knows the client or any other witness has already committed perjury, the lawyer must: i. ii. attempt to persuade the client to recant if that fails, disclose the perjury to the tribunal iii. then withdraw (if there is an irreconcilable conflict), with court approval c. if the lawyer learns of the perjury after the conclusion of the proceedings (including all appeals), the lawyer is no longer obligated to reveal it d. if the lawyer reasonably believes (but does not know) the client or another witness intends to testify falsely, the lawyer may refuse to offer the testimony, unless it is the testimony of a criminal defendant (which the lawyer should offer unless the lawyer knows it is false)

38

7. a lawyer shall not seek to illegally influence (e.g., bribe, threaten) a judge, juror, or prospective juror; such acts are usually crimes, such as obstruction of justice a. example: ex parte contact with such persons (except where authorized -- TRO hearing); contact with a juror about a case is always improper, regardless of who initiates or whether the lawyer is trial counsel b. a lawyer shall have NO ex parte contact with a juror prior to or during trial (e.g., use different elevators, eat in different restaurants) i. an attorney may hire a jury consultant to perform general demographic research about members of the jury pool, but the consultant should have no direct contact with the jurors or their families ii. after trial, a lawyer may contact a juror unless (1) the contact is prohibited by law or court order; (2) the juror has made known to the lawyer a desire not to communicate; or (3) the communication involves misrepresentations, harassment, etc. MPRE Question 34 c. conduct intended to disrupt the tribunal (lawyer feigns a stroke to obtain a mistrial) is also improper 8. a trial lawyer (and anyone acting on his or her behalf) shall avoid making extrajudicial statements that are likely to prejudice proceedings (particularly in jury trials); prosecutors have a duty to take care to prevent others (e.g., the police) from making improper extrajudicial statements a. proper statements: basic facts in public record (names of parties, charges, claims, defenses, requests for assistance in obtaining evidence, warning of danger of fugitives, facts about arrest) b. improper statements (usually more of a concern in criminal cases, particularly those tried before a jury): character or reputation of parties or witnesses; the identity or expected testimony of witnesses; confessions and pleas; polygraph tests; the existence of physical evidence; opinions on guilt or innocence; inadmissible evidence; the fact that the defendant has been charged

39

with a crime (unless accompanied by statement that defendant is presumed innocent) c. Mitigation Rule: a lawyer may disclose otherwise improper information if necessary to mitigate recent adverse publicity (e.g., defense counsel may state that the accused was coerced to confess after the prosecution has told the press that the accused has confessed) 9. a lawyer shall not testify in a case in which he or she is a trial lawyer, except where: a. the testimony relates to an uncontested issue

MPRE Question 44 b. the testimony relates to the nature and value of the legal services (e.g., lawyer testifies in connection with a fee petition in a civil rights case or class action) c. disqualification would work a substantial hardship on client (e.g., lawyer is called as a surprise witness by an opponent late in the proceedings) Note: this prohibition is not imputed to other lawyers in the trial lawyers firm, unless the lawyers testimony would be adverse to the clients testimony 10. a lawyer who knowingly presents false or perjured evidence would not only be subject to discipline and litigation sanctions, but may be subject to criminal liability B. A lawyers duty to opposing counsel 1. A lawyer shall not: a. unlawfully obstruct another partys access to evidence (but, except for prosecutors, lawyers have no affirmative duty to disclose information to an opponent; lawyers generally are obligated to disclose information only if there has been a proper discovery request) i. example: lawyer destroys relevant documents after receiving a discovery request seeking such documents. Such spoliation violates the Rules, FRCP 37, and possibly criminal obstruction of justice statutes. The same would hold

40

true for destruction of documents the lawyer knows or should know are material to litigation before a discovery request is served (e.g., lawyer orders client to destroy relevant documents upon receiving complaint but prior to discovery). ii. a lawyer shall not make frivolous discovery requests and must act diligently to respond to discovery iii. if a lawyer fails to fully comply with a valid discovery request, the lawyer and/or the client and/or the lawyers firm (note that there is some authority for the proposition that a court may not impose FRCP sanctions against a law firm) may be subject to monetary sanctions under FRCP 37. The court may also impose such sanctions as striking evidence or witnesses, dismissing all or part of a plaintiffs case, entering default judgment against a defendant or dismissing all or some of the defendants defenses, or holding the client and/or the lawyer in contempt. Contempt is usually a sanction of last resort. There are two types of contempt: civil and criminal. Civil contempt (fines or imprisonment) is imposed to coerce a party or attorney to comply with a court order. Criminal contempt, which is rarely imposed in a discovery dispute, is a separate criminal proceeding for a past violation of a court order. FRCP 11 does NOT apply to discovery.

iv.

b. falsify or tamper with evidence or assist another in doing so (also violates FRCP 37 and obstruction of justice statutes) c. offer an inducement to a witness to testify, except: i. a lawyer may pay the reasonable travel expenses of lay and expert witnesses ii. a lawyer may reimburse a lay witness for lost wages/time expended (i.e., the amount the witness lost by testifying) MPRE Question 42

41

iii. a lawyer may pay an experts fee for testifying, but the fee cannot be contingent on the outcome of the case d. knowingly disobey a court order, except for an open refusal that no valid obligation exists i. such conduct may also subject the lawyer to contempt; a lawyer is not obligated under the Rules to be held in contempt for a client MPRE Question 48 e. at trial, a lawyer shall not allude to irrelevant or inadmissible information to inflame the jury i. examples: lawyer asserts personal knowledge of facts (unless testifying); lawyer states a personal opinion on the justice of a cause or the credibility of a witness (In my opinion, W is lying) or the liability of a civil litigant or the guilt or innocence of an accused f. a lawyer shall not request that a person (including a client) leave the jurisdiction to avoid giving evidence; in addition, a lawyer shall not request that a person refrain from voluntarily giving evidence to another party, except when: i. the person is the lawyers client or ii. the person is a relative or employee of the lawyers client and the lawyer believes that the persons interests will not be adversely affected by such advice (e.g., it is usually proper for corporate counsel to advice corporate employees not to speak to opposing counsel) C. A lawyers special duties as a prosecutor 1. Prosecutors: a. shall not bring charges unsupported by probable cause Example: The public is demanding an arrest and conviction in a series of rapes. L files charges against D to quiet the public. L does not believe that D is the rapist, but brings the charges anyway. L is subject to discipline.

42

b. shall make reasonable efforts to ensure that the accused has been given his Miranda warnings and has been given counsel c. shall not seek to obtain waivers of important rights from unrepresented defendants d. shall timely disclose all exculpating and mitigating information to the defense (unlike other lawyers, prosecutors have an affirmative duty to disclose adverse information to opponents) MPRE Question 49 e. shall not subpoena lawyers to testify about former or current clients, except where the information is not privileged, is essential, and there is no alternative source duties in non-adjudicative proceedings (e.g.,

D. A lawyers legislative hearings)

IV.

1. a lawyer may appear on behalf of and testify for a client at official legislative or administrative hearings, but the lawyer must disclose that he or she is appearing in a representative capacity (e.g., a lawyer may testify for client before the FTC, but must inform the FTC that he or she is there on behalf of a client) and the lawyer must comply with all of the Rules (e.g., prohibition against offering false testimony) relating to the lawyers duties to a tribunal and opposing parties A Lawyers Duty to Third-Parties A. Truthfulness 1. a lawyer shall not knowingly make false statements of fact or law to third parties a. Exception: puffing in negotiations (My clients land is worth twice that amount or My client cannot accept any settlement below $1 million) MPRE Question 30 2. a lawyer must disclose relevant facts necessary to avoid assisting a client in fraud or a crime, unless such information is protected by Rule 1.6 (which is often the case); if the information is protected by Rule 1.6 and the client insists that lawyer not disclose a fact even though substantive law requires disclosure,

43

the lawyer should resign (and the lawyer may be allowed to reveal the information if it fits within one of the exceptions to Rule1.6) TIP: a lawyer generally is not required to disclose facts adverse to the lawyers client (e.g., defendants lawyer is not obligated to voluntarily inform court that defendant has prior convictions) B. Communicating with Represented Persons 1. General Rule: lawyer may not communicate with a represented person about the subject of the representation, unless authorized to do so by law or court order (this rule applies even though the communications are initiated by the represented person) 2. The rule includes all persons represented by counsel, not just adverse parties, and the officers, managers, and employees (whose actions or statements may be imputed to the entity) of a represented corporation or other entity 3. The rule applies only to persons the lawyer knows or should know are represented 4. The rule applies only to communications related to the subject of the representation a. Exceptions to the rule: i. ii. persons counsel consents (never happens) client to client communications are fine iii. communication does not relate to subject of representation (e.g., lawyer is suing IRS for client; lawyer may still contact IRS about her own tax returns) Example: L represents A in A v. B. B is represented by W. While traveling on vacation, L is seated next to B on an airplane. L asks to move to another seat, but no other seats are available. L and B make small-talk during the flight about the weather, etc. L is not subject to discipline. MPRE Question 21 C. Dealings with Unrepresented Persons

44

1.

The lawyer must not: a. state or imply that he or she is disinterested (e.g., I dont care who wins; I only want to find out the truth) b. fail to correct a persons misunderstanding of the lawyers role; if the lawyer knows or should know of the misunderstanding, the lawyer must make clear that he or she is not disinterested and that he or she is not the lawyer of the third party c. the only advice the lawyer may give the third person is to obtain counsel; no other legal advice may be given to third persons (this is particularly true if the advice is favorable to the lawyers client)

2. The lawyer must not use means that have no substantial purpose other than to embarrass, delay, or burden a third person, or use methods of obtaining evidence that violates the third persons legal rights (e.g., lawyer bugs witnesss home; lawyer threatens witness). 3. A lawyer who receives a document from a third party by mistake (e.g., an opposing attorney inadvertently faxes a privileged document) must promptly notify the sender. V. A Lawyers Duty to Other Members of the Lawyers Firm A. Negligent Supervision (fact-sensitive application) 1. A partner (or lawyer with comparable managerial responsibility) in a law firm may be disciplined for failure to ensure that the firm has measures in place to assure that all lawyers and other employees in the firm conform to the Rules (measures may be informal in small firms, but may require CLEtype efforts in larger firms) 2. A supervising lawyer may be disciplined for failure to make reasonable efforts to ensure that a junior lawyer or employee of the firm conforms to Rules

B.

Vicarious Liability 1. A lawyer may be disciplined for another lawyers or employees violation of the Rules if:

45

a. b.

the lawyer orders the conduct with knowledge of the conduct, the lawyer ratifies it c. for partners or comparable supervising lawyers, the lawyer knows of the conduct when its consequences could be mitigated or avoided, but fails to take reasonable remedial measures (e.g., paralegal improperly places client funds in law firms office account; partner learns of mistake and corrects it; no vicarious liability, but partner is possibly liable for failure to ensure that paralegal was properly trained)

Note: a law firm, as opposed to partners and supervising lawyers, has no vicarious (disciplinary) liability for the acts of its lawyers or employees, but it may have such liability in tort C. Liability of Junior Lawyers 1. General Rule: Junior lawyer is just as liable as senior lawyer a. exception: if junior lawyer acts in accordance with a supervising lawyers reasonable resolution of an arguable ethical question, junior lawyer is not liable (e.g., issue involves whether a former clients matter is substantially related to that of a current client; issue is a close call; senior lawyer believes there is no conflict; junior lawyer so proceeds; no liability for junior lawyer, but supervising lawyer remains liable). D. A Lawyers Business Relationship with Non-Lawyers 1. General Rule: a lawyer may not share fees with a non-lawyer a. Exceptions: payment time ii. profit-sharing (compensation and retirement) plans for non-lawyer employees of firm iii. a lawyer may share courtawarded fees with a nonprofit organization that employed or recommended the lawyer to a deceased lawyers estate for

i. reasonable

46

2. A lawyer may not enter into a partnership with a nonlawyer if the partnership will perform any legal practice (but a lawyer may enter into non-legal business with a non-lawyer) 3. A lawyer may not practice law in a corporation or other entity if a non-lawyer owns any interest in the entity (i.e., a shareholder), if a non-lawyer is an officer or director (or a person with similar responsibility) of the entity, or if a non-lawyer has the right to control the professional judgment of a lawyer (e.g., WalMart could not start a law firm because it has non-lawyer shareholders, officers, directors, and general managers) MPRE Question 12 4. If someone other than a client is paying the attorneys fees (e.g., auto insurer) or referring the client (e.g., the clients parent or employer), that person may not direct or control the professional judgment of the lawyer and is not entitled to confidential information about the client E. Unauthorized Practice of Law (UPL) 1. State law defines practice of law; e.g., appearing in court as a lawyer, drafting wills and trusts a. The completion of tax returns and the filling in of blanks on forms, such as deeds or powers of attorney, is generally not considered the practice of law.

2. It is a violation of rules for an attorney to engage in UPL (e.g., appearing in a court, including a federal court, in which attorney is not admitted to practice permanently or pro hac vice) a. pro hac vice is permission by a court for an attorney licensed in another jurisdiction to appear in that court as a lawyer for one case; pro hac vice approval is within the absolute discretion of the court; a lawyer may prepare for a proceeding in another jurisdiction if the lawyer reasonably expects to be admitted pro hac vice 3. a lawyer who is not admitted to practice in a jurisdiction shall not: a. b. establish an office or other systematic and continuous presence in the jurisdiction; or hold out to the public or otherwise represent that the lawyer is admitted to practice law in the jurisdiction.

47

4.

it is not UPL: a. for an in-house or government lawyer to represent his or her employer (or a related entity) in matters in jurisdictions in which the lawyer is not licensed (except for appearances in court) Example: L is an in-house lawyer for ABC Corp. L is licensed in New York. ABC transfers L to its Iowa office to serve as General Counsel. L may represent ABC in Iowa, despite the fact that L is not licensed there. L may not, however, appear in Iowas courts. b for a lawyer to act with respect to a matter in another jurisdiction that is reasonably related to the lawyers representation of a client in a jurisdiction in which the lawyer is licensed Example: A is licensed in Illinois and represents C in litigation in Illinois. A may travel to Indiana to take a deposition for that case and to Nevada to inspect documents related to that case, even though A is not licensed in either Indiana or Nevada. Example: A is licensed in Illinois and represents C, an Illinois corporation. C wishes to enter into a contract with D, a California corporation. A may travel to California to negotiate the contract and inspect Ds records. c. if the lawyer associates with a lawyer licensed to practice in that jurisdiction and that other lawyer actively participates in the matter Example: A is asked to negotiate a contract for Client, who is located in California. A is licensed in New York. A may represent Client if A associates with a California lawyer that will be actively involved in the negotiation. d. for a lawyer to represent a client in alternative dispute resolution in this jurisdiction (in which the lawyer is not licensed) if the ADR reasonably relates to a matter in a state in which the lawyer is licensed e. a lawyer who temporarily practices subject to the preceding paragraphs is subject to the jurisdiction of the disciplinary authority of the state in which the lawyer is so practicing.

48

5. It is a violation of the Rules for an attorney to assist others (e.g., non-lawyers or disbarred lawyers) in UPL (e.g., instructing non-lawyers in trust department of a bank to draft wills for others so that they can draft such wills without the supervision of a lawyer) a. Exceptions: A lawyer may: i. ii. counsel non-lawyers who wish to proceed pro se provide general legal instruction to non-lawyers (e.g., teaching a seminar for real estate agents) iii. use paralegals or other firm employees to draft documents, etc. under the lawyers supervision MPRE Question 8 F. Restrictions on a Lawyers Right to Practice 1. a covenant-not-to-compete (or any other restriction on a lawyers right to practice after leaving the firm) constitutes a violation by all parties to the contract a. exception: (1) a firm may condition retirement benefits on non-competition, and (2) a lawyer buying another lawyers firm may restrict the selling lawyers right to compete, if such restrictions are reasonable in time and scope

2. a settlement agreement may not restrict a lawyers right to practice (i.e., a clause prohibiting plaintiffs lawyer from representing any other clients in litigation against defendant); such agreements constitute a violation by all lawyers to contract G. If a law firm offers non-legal services (e.g., title insurance, accounting, trust services), the law firm generally must comply with the Rules as to all services

VI.

A Lawyers Duty to the Public (Pro Bono, Court Appointments, Legal Aid Organizations, and Law Reform Activities) A. Pro Bono 1. Aspiration (a non-binding professional responsibility): per year 50 hours

49

a. a substantial amount of these hours should be comprised of free work for indigents, charities, etc. or substantially-reduced-fee work for low income clients or charities, or participation in legal reform activities b. lawyers may also satisfy the pro bono requirement by making donations to legal aid organizations (in those years where the lawyer is too busy to provide 50 hours) c. B. a lawyer will NOT be subject to discipline for failure to comply with 50-hour requirement

Accepting Clients (including Judicially-Appointed Clients) 1. General Rule: A lawyer may decline to represent any client (i.e., a lawyer is not a bus; he or she does not have to pick up all passengers) a. Exception: a lawyer shall not avoid courtappointed representation unless the: i. representation is likely to result in a violation of Rules or law (e.g., client intends to commit perjury) ii. representation is likely to result in unreasonable financial burden on lawyer (e.g., a new lawyer with limited resources being asked to undertake lengthy litigation for free) iii. client or case is repugnant to lawyer (e.g., requiring a Jewish lawyer to represent a Nazi)

C.

Legal Services Organizations (LSO) 1. A lawyer may serve as an officer, director, or member of an LSO (while simultaneously practicing law in a private firm) even though the LSO represents persons with interests adverse to a client of the lawyer a. Exception: the lawyer shall not knowingly participate in an LSO decision or action incompatible with the lawyers obligations to a client (e.g., lawyer shall not participate in LSOs decision to sue a client of lawyer)

D.

Law Reform Activities (LRA)

50

1. A lawyer may serve as an officer, director, or member of an organization involved in LRA while simultaneously practicing law in a private firm. 2. If an LRA will benefit a private client of the lawyer, the lawyer shall disclose this fact, but need not disclose the identity of the client Example: Lawyer is a director of the Practicing Law Association (PLA), an organization that advocates tort reform. The PLA lobbies state legislatures to adopt reform laws. The PLA is considering adopting a rule setting a cap on punitive damages in personal injury cases. Lawyers client, Allhands Insurance Company, informs Lawyer that it favors such a rule. Lawyer can vote in favor of the rule if Lawyer informs the PLA that the rule will benefit one of Lawyers clients (although Lawyer does not have to identify the client by name). E. Non-Profit Legal Services Programs 1. A lawyer who, under the auspices of a nonprofit organization or court, provides short-term limited legal services (e.g., telephone advice line) to a client is subject to Rules 1.7 (concurrent conflicts), 1.9 (former client conflicts), and 1.10 (imputed disqualification) only if the lawyer knows of a conflict of interest. As a general rule, Rule 1.10 will not apply to such representation. VII. A Lawyers Duty to the General Public (Advertising and Solicitation) A. Definitions: 1. Advertising: promotions aimed at large groups, usually the general public; examples: television, newspaper, radio, internet ads, recorded telephone messages, mass mailings, yellow page ads, magazines, billboards, and probably spam email 2. Solicitation: promotions targeted to a particular person, often one that the attorney knows is in need of a particular legal service; examples: live telephone calls; in-person solicitation; targeted, direct-mail; and real-time electronic contacts B. Advertising 1. General Rule: Ads are permissible as long as they are not false, deceptive or misleading; First Amendments commercial speech rules apply; a state may regulate ads only if it has a

51

substantial interest and the regulation is no broader than necessary; the only substantial interests recognized by courts are the prevention of false, misleading, and potentially misleading ads Example: L is a licensed attorney. L runs the following advertisement in the local paper: My name is L. I am an attorney and a C.P.A. I handle medical malpractice and personal injury cases. I graduated first in my class from Appalachian School of Law in 2003, and I have a degree in accounting from the University of Tennessee. I was raised in Knoxville, and speak fluent Spanish. If you have a legal problem, please call me or send me an email. Your initial consultation is free. My office accepts credit cards. Is L subject to discipline? Answer: no, as long as all statements in the advertisement are true. Example: L is a licensed attorney. L runs an advertisement in the local newspaper stating truthfully: My name is L. I am a new attorney. As many of you know, my brother and father are circuit court judges in town. Please call me should you need any legal assistance. This is probably improper because it implies that L has the ability to improperly influence judicial officials. a. taste, decorum, dignity, tradition, etc. are not substantial interests sufficient to ban ads b. a comparison to other lawyers that cannot be verified by hard facts (I am the best lawyer in town) or that is potentially misleading (I have never lost a jury trial -- of course, Ive tried only one) are impermissible i. testimonials may be used if the information makes clear that the results different (i.e., disclaimer), and consents to the use of her name of former clients is true, the lawyer of each case are the former client

c. factual information about an attorney (e.g., college and law school attended, languages spoken, credit cards accepted, fees charged, other degrees [C.P.A., M.D.], length of residence in community, names of references) may be included in ads if true d. ads must include the name and office address of at least one lawyer or the law firm C. Solicitation

52

1. General Rule: In-person, live telephone solicitation, and real-time electronic contact (by the lawyer or anyone acting on the lawyers behalf) is prohibited a. Exceptions: i. Lawyer has prior professional (probably means attorney-client) relationship with prospective client MPRE Question 4 ii. prospective client and lawyer are family members iii. lawyers primary motive is not pecuniary gain (i.e., lawyers may solicit pro bono clients) iv. 2. the person solicited is a lawyer

General Rule: Targeted, direct-mail solicitation permitted a. Exceptions: i. false and misleading mail prohibited ii. prospective client lawyer not to send letters (rarely occurs) iii. has told

letter involves coercion, duress, harassment iv. although the Model Rules contain no such restriction, the Supreme Court has upheld a states ban on targeted, direct-mail solicitation in personal injury/wrongful death cases within 30 days of the accident

b. targeted-direct mail solicitation and recorded telephone messages must contain the words Advertising Material on envelope or at beginning and end of tape i. such words are not necessary if lawyer has prior professional or family relationship with prospective client c. a lawyer may be a member of a prepaid legal services plan which solicits prospective members in-person or by live telephone calls

53

i. the lawyer may not have an ownership or management interest in the plan, nor may the lawyer personally participate in the solicitation ii. the plan may not solicit persons who it knows are in need of legal services in a particular matter D. Communicating Fields of Practice 1. Permissible a. b. c. d. Practice limited to .... We handle ..... Jones & Smith. Personal injury, medical malpractice ..... Patent Lawyer or Admiralty Lawyer, where true e. Certified as a Trial Specialist by the State of Ohio, where state certifies lawyers (facts of question would have to provide this info) f. Certified as a Trial Specialist by National Association of Trial Attorneys i. must be true and the certifying organization must be approved by the state bar or the ABA 2. Impermissible a. b. E. We specialize in ..... Experts in .......

Firm Names and Letterheads 1. no false or misleading names may be used 2. trades names (The ABC Law Firm) are permissible as long as such names do not imply a connection with government (Greensboro Law Firm) or with a public or charitable legal organization (Red Cross Legal Clinic); if such implications are made, a disclaimer (Not associated with the Red Cross) would

54

be required. A lawyer or firm may also use a distinctive website address (as long as its not misleading). 3. a multistate law firm may use the same name (Sidley & Austin) in each state as long as its letterhead clearly identifies lawyers not licensed in the state where the letterhead is being used: Example: Firms New York letterhead provides
Smith, Jones & Davis Offices in New York, Chicago, and Los Angeles Ms. Jones is licensed to practice law in California Mr. Davis is licensed to practice law in Illinois

This is appropriate (assuming all information is true). 4. used 5. if lawyer leaves a firm to assume public office for a substantial period, lawyers name must be removed a. partner Jones is elected as a full-time judge; Jones name must be removed b. partner Jones is appointed to fill a judicial vacancy for two weeks; Jones name need not be removed c. partner Jones is appointed as a part-time judge but continues to practice with firm in accordance with state law; Jones name need not be removed 6. law firm names may not imply a partnership or association where none exists a. A and B share office space but are not partners. A and B cannot use letterhead, signs, etc. implying a partnership (A and B, Attorneys at Law, would imply a partnership) 7. two firms may claim that they are affiliated or associated if they have a regular, ongoing relationship (but this may create additional conflicts of interest) a. Jones & Smiths letterhead states affiliated with Lopez and Sanchez, Mexico City; this is proper if there is an ongoing relationship between firms the names of deceased and retired partners may continue to be

55

F.

Referral Fees, etc. 1. General Rule: A lawyer may not pay anyone to refer clients (e.g., lawyer pays a runner $1,000 for each referral; lawyer pays real estate agent 10% fee for referrals; lawyer gives accountant a gift for years of referrals) a. Exceptions: i. a lawyer may pay a referral fee to another lawyer if the overall fee to the client is not increased, client agrees in writing to the fee division, and both lawyers assume responsibility (i.e., malpractice liability) for representation (supra) ii. a lawyer may pay for ads like any other business iii. a lawyer may pay the regular fee of a non-profit or approved lawyer referral service iv. a lawyer may purchase a law firm, including its clients, subject to the limitations (supra) v. a lawyer may enter into a nonexclusive reciprocal referral agreement for specified time periods with another lawyer or a non-lawyer professional; clients obtained through such agreements must be informed of the nature of such agreement and the agreement must not interfere with the lawyers professional judgment 2. A lawyer may not make political contributions so as to obtain legal engagements or appointments a. Example: ABC Law Firm donates $50,000 to the election campaign of Judge X. ABC rarely makes political donations. Judge X is elected and immediately appoints ABC to handle all guardianships for the court. In such case, an inference arises that ABC bought the appointments.

56

VII.

A Lawyers Duty to the Profession A. Bar Applications 1. Bar applicants must not make false statements or fail to disclose relevant information to the bar; if such misrepresentations are discovered prior to admission, the application may be denied; if such misrepresentations are discovered after admission, the lawyer would be subject to discipline under the Rules. MPRE Question 2 2. Attorneys asked to provide information about bar applicants (i.e., references) shall not make false statements and shall only provide information that they know is true. Attorneys also have an affirmative duty, subject to the next paragraph, to correct any misapprehension known to the lawyer to have arisen in the application process. 3. Attorneys asked to provide information about bar applicants (i.e., references) shall disclose relevant information about an applicant, unless such information is protected by Rule 1.6 (e.g., attorney represents the applicant in the application process), and shall not fail to respond to a lawful demand for information B. Disciplinary Proceedings 1. Attorneys, including the attorney who is the subject of the disciplinary proceeding and attorney-witnesses, shall not make false statements, fail to disclose information, or fail to respond to a lawful demand for information in a disciplinary proceeding a. Exceptions: attorneys may assert the 5th Amendment, where appropriate; and attorneys are not required to disclose information protected by Rule 1.6 (e.g., where the attorney represents the subject of disciplinary proceedings)

57

C. Officials

Statements Concerning Judges and Other Legal 1. Attorneys may not make statements about judges, other legal officials, or candidates for judicial office that the attorney knows to be false or with reckless disregard for the truth

2. Attorneys statements about judges 3. the CJC.

may,

however,

make

truthful

A lawyer seeking judicial office must comply with

D.

Duty to Report Professional Misconduct 1. A lawyer who knows that another lawyer or judge has committed a violation of the Rules or the law that raises a substantial question as to the lawyers or judges honesty, trustworthiness, or fitness must report the lawyer or judge to appropriate authority (e.g., state bar) a. Exception: There is no duty to report when the information is protected by Rule 1.6, which is often the case, unless the client consents to disclosure (the lawyer should encourage the client to consent when the disclosure will not prejudice the client); in addition, there is no duty to report if the lawyer learned the information as part of an approved lawyers assistance program (e.g., state bar approved substance abuse counseling for lawyers). MPRE Question 24 b. a lawyer may not threaten to report a disciplinary matter to obtain an advantage in litigation

58

E.

Misconduct (not while representing a client) 1. A lawyer may be disciplined for personal activities (i.e., activities unrelated to the representation of a client), including: a. violating, attempting to violate, or using another person to violate the Rules (i.e., a violation of the Rules is, in fact, a double violation) b. a criminal act that reflects adversely on the lawyers honesty, trustworthiness, or fitness to practice (e.g., tax evasion, but not a speeding ticket, unless the lawyer has dozens of speeding tickets). i. a lawyer does not violate this rule if the lawyer in good faith believes that no valid obligation exists Example: L is involved in a peace protest at city hall. The police ask the protesters to leave. L believes that the First Amendment protects his right to picket on government sidewalks. The police arrest L and he is convicted of disorderly conduct. If L in good faith believed that he was legally entitled to picket on the sidewalk, L would not be subject to discipline (even if L was mistaken as to the law). c. dishonesty, fraud, etc. (e.g., filing a false

insurance claim) MPRE Question 3 d. conduct prejudicial to administration of justice (e.g., blatant discrimination in connection with the representation of a client) e. imply an ability to improperly influence a government official (e.g., telling client that the judge can be bribed

59

or that the mayor owes you a favor in exchange for a political contribution) or imply an ability to achieve results by means that violate the Rules or other law Example: L is a licensed attorney. L runs an advertisement in the local newspaper stating truthfully: My name is L. I am a new attorney. As many of you know, my brother and father are circuit court judges in town. Please call me should you need any legal assistance. This is probably improper because it implies that L has the ability to improperly influence judicial officials. The same may hold true if L simply states these facts to a prospective client. MPRE Question 32 f. knowingly assist a judge in violating Rules (e.g., lawyer and judge engage in improper ex parte communications). F. Lawyers? Which State Has Jurisdiction to Discipline

1. Each and every jurisdiction in which a lawyer is licensed may discipline lawyer for same conduct (e.g., lawyer is licensed in VA and TN; lawyer found guilty of perjury in New York; TN and VA may discipline lawyer); a lawyer may also be disciplined by a jurisdiction in which she is not licensed if she provides or offers to provide legal services in that jurisdiction. G. Whose Law Applies? 1. If violation occurs in litigation, the law of the state in which the court sits governs. 2. In other cases: a. the rules of the jurisdiction in which the conduct occurred governs or

60

b.

if the predominant effect of the conduct is in a different jurisdiction, that jurisdictions rules govern; a lawyer is not subject to discipline if the lawyers conduct conforms to the laws of a jurisdiction in which the lawyer reasonably believes the predominant effect of the lawyers conduct will occur. Example: A is licensed to practice in State 1. State 1's rules do not prohibit inperson solicitation of clients in personal injury cases. Client lives in State 2, but was injured in State 1. State 2 prohibits in-person solicitation of clients in personal injury cases. A telephones Client offering to represent Client in a personal injury suit in State 1. A reasonably believes that the rules of State 1 govern his conduct. Even if the predominant effect of As conduct is in State 2, A should not be subject to discipline because he reasonably believed that the rules of State 1 governed his conduct.

THE DUTIES OF A JUDGE


Governing Law : The ABAs Model Code of Judicial Conduct (CJC) governs the discipline of judges. Anyone, whether or not a lawyer, who is an officer of a judicial system and who performs judicial functions, including magistrates, court commissioners, special masters, or referees, are judges within the meaning of the CJC. Lawyers who are candidates for judicial office are also subject to the CJC. Retired judges and part-time judges are exempt from many of the provisions of the CJC, particularly those restricting outside activities (e.g., practicing law). Judges are generally immune from civil liability for official acts, but may be criminally prosecuted like any other citizen. I. A Judge Shall Uphold the Integrity and Independence of the Judiciary A. a judge should be impartial and independent

61

B.

a judge must comply with the law (e.g., repeated and flagrant refusal to comply with decisions of higher courts may subject the judge to discipline) a judge must act with integrity in his or her personal life (e.g., a judge may be disciplined for violating criminal laws or committing fraud)

C.

II. A Judge Shall Avoid Impropriety and the Appearance of Impropriety A. a judge shall not voluntarily testify as a character witness (a judge must comply with a subpoena to so testify, but should discourage a party from calling him or her as a character witness) a judge shall not allow family, social, political, or other relationships to influence his or her judicial conduct a judge shall not use his or her status to gain personal advantage (e.g., attempting to avoid traffic tickets when stopped by the police or using judicial letterhead in his or her private business to attract clients or collect debts) a judge must avoid lending the prestige of judicial office for the advancement of the private interests of others (e.g., a judge must not use the judge's judicial position to gain advantage in litigation involving a member of the judge's family). Example: Judges son is on trial for shoplifting in neighboring town. Judge should have no contact with presiding judge. E. a judge may, based on the judge's personal knowledge, serve as a reference or provide a letter of recommendation. However, a judge must not initiate communication to a sentencing judge or a probation or corrections officer, but may provide to such persons information for the record in response to a formal request. Example: Judge may write a letter of reference for her former law clerk so that the clerk may obtain a position with a private firm.

B. C.

D.

62

F.

A judge shall not hold membership in any organization that currently practices invidious discrimination on the basis of race, sex, religion, national origin, or any other class protected by state law. 1. A judge may, however, hold membership in an organization dedicated to the preservation of religious, ethnic or cultural values of legitimate common interest to its members, or that is in fact and effect an intimate, purely private organization whose membership limitations could not be constitutionally prohibited. Absent such factors, an organization is generally said to discriminate invidiously if it arbitrarily excludes from membership on the basis of race, religion, sex or national origin persons who would otherwise be admitted to membership.

63

Examples: a. Judge is a member of the NAACP or the IrishAmerican Scholarship Committee, groups which seek to promote the ethnic or cultural heritage of its members. This is appropriate. b. Judge plays poker with six men every Wednesday night. Women are not welcome. This is appropriate as a purely private organization. c. Judge belongs to the KKK or is a member of a country club that excludes Jews, etc. based purely on the persons ethnicity or religion. These are inappropriate activities. 2. In addition, it is a violation for a judge to arrange a meeting at a club that the judge knows practices invidious discrimination on the basis of race, sex, religion or national origin in its membership or other policies, or for the judge to regularly use such a club. Example: Judge golfs twice a week at Oak Hills Country Club, but Judge is not a member of the club. The club excludes African-Americans. Judge is subject to discipline. 3. a public manifestation by a judge of approval of invidious discrimination on any basis gives the appearance of impropriety. Example: Judge is not a member of the KKK, but publicly supports the group. This is inappropriate. 4. If a judge learns that an organization to which the judge belongs engages in invidious discrimination, the judge is permitted, in lieu of resigning, to make immediate efforts to have the organization discontinue its invidiously discriminatory practices, but is required to suspend participation in any other activities of the organization. If the organization fails to discontinue its invidiously discriminatory practices as promptly as possible (and in all

events within a year of the judge's first learning of the practices), the judge is required to resign immediately from the organization. III. A Judge Shall Act Impartially and Diligently A. The judicial duties of a judge take precedence over all the judge's other activities. A judge should avoid any activity that substantially interferes with the judges judicial duties. Example: A is a full-time judge. A is in need of money and is considering buying a business in which A will be required to work 20 hours per week. This is inappropriate. B. A judge shall hear and decide matters assigned to the judge except those in which disqualification is required. 1. A judge shall dispose promptly of the business of the court; continuances should be granted only upon good cause A judge should maintain professional competence through CLE or otherwise.

2. C. D.

A judge shall require order and decorum in proceedings before the judge. A judge shall be patient, dignified, and courteous to litigants, jurors, witnesses, lawyers and others with whom the judge deals in an official capacity, and shall require similar conduct of lawyers, and of staff, court officials and others subject to the judge's direction and control. A judge shall perform judicial duties without bias or prejudice (and prevent others from doing so). A judge shall not, in the performance of judicial duties, by words or conduct, manifest bias or prejudice or sexual harassment. A judge shall accord to every person who has a legal interest in a proceeding, or that person's lawyer, the right to be heard.

E.

F.

65

G.

A judge shall not initiate, permit, or consider ex parte communications, except: 1. ex parte communications for scheduling, administrative purposes or emergencies that do not deal with substantive matters or issues on the merits that are authorized by law (but should be used sparingly) Example: L represents A in the case of A v. B. L calls Judge to schedule a hearing for a motion for summary judgment. L then notifies Bs attorney of the date. This is not improper, but it would have been better if both attorneys had called Judge. Example: L files a motion for an emergency TRO. L attempts to notify her opponent, but to no avail. Local law allows judges to grant TROs ex parte. Judge grants the TRO after an ex parte hearing. Judge is not subject to discipline. 2. A judge may obtain the advice of a disinterested expert on the law applicable to a proceeding before the judge if the judge gives notice to the parties of the person consulted and the substance of the advice, and affords the parties reasonable opportunity to respond. It would be better for the judge to encourage such persons to file an amicus brief. A judge may, without notice to the parties, confer with court personnel, other judges in the court, and the judges law clerks.

MPRE Question 33 3. A judge may, with the consent of the parties, confer separately with the parties and their lawyers in an effort to mediate or settle matters pending before the judge.

H.

A judge must not independently investigate facts in a case and must consider only the evidence presented. Example: Judge is presiding over As personal injury trial. Judge believes A is faking his injuries. Judge asks her clerk to follow A during a lunch break to determine whether A is faking. This is inappropriate.

66

Example: X is a witness. After both the prosecution and defense have questioned X, Judge asks X a couple of questions to clarify an issue. This is appropriate. I. A judge may request a party to submit proposed findings of fact and conclusions of law, so long as the other parties are apprised of the request and are given an opportunity to respond to the proposed findings and conclusions. Note: This is a common practice in bench trials. The judge often assigns this task to the prevailing party. This is appropriate, if the other parties are given a chance to object to and supplement the findings. J. A judge should encourage and seek to facilitate settlement, but parties should not feel coerced into surrendering the right to have their controversy resolved by the courts. A judge shall not, while a proceeding is pending or impending in any court, make any public comment that might reasonably be expected to affect its outcome or impair its fairness or make any nonpublic comment that might substantially interfere with a fair trial or hearing. The requirement that judges abstain from public comment regarding a pending or impending proceeding continues during any appellate process and until final disposition. This Section does not prohibit a judge from commenting on proceedings in which the judge is a litigant in a personal capacity, but in cases such as a writ of mandamus where the judge is a litigant in an official capacity, the judge must not comment publicly. A judge may also make public statements explaining the judicial process. Example: Judge is hired by CNN to comment on highprofile cases. This is inappropriate even though Judge has no involvement in the cases, if the Judges comments might reasonably be expected to affect the outcome of such cases. L. A judge shall not commend or criticize jurors for their verdict other than in a court order or opinion in a proceeding, but may express appreciation to jurors for their service to the judicial system and the community.

K.

67

Example: Judge informs jurors at the end of the case: I think you have reached the right decision. This is inappropriate. M. A judge shall not disclose or use, for any purpose unrelated to judicial duties, nonpublic information acquired in a judicial capacity. Example: Judge is presiding over a patent dispute between A and XYZ Corp. During the trial, Judge learns in camera that XYZ Corp. will soon announce a cure for cancer. The next day, Judge purchases XYZ Corp. stock. Judge is subject to discipline. N. O. A judge shall diligently discharge administrative responsibilities the judge's

A judge shall not make unnecessary appointments (e.g., guardians, masters, etc.). A judge shall exercise the power of appointment impartially and on the basis of merit. A judge shall avoid nepotism and favoritism. A judge shall not approve compensation of appointees beyond the fair value of services rendered. A judge who receives information indicating a substantial likelihood that another judge has committed a violation of the CJC shall take appropriate action. A judge who receives information indicating a substantial likelihood that a lawyer has committed a violation of the Rules that raises a substantial question as to the lawyer's honesty, trustworthiness or fitness as a lawyer in other respects shall take appropriate action. Appropriate action may include direct communication with the judge or lawyer who has committed the violation, other direct action if available, and reporting the violation to the appropriate authority or other agency or body. A judge who knows that another judge or lawyer has committed a violation of the CJC or Rules that raises a substantial question about the lawyers or judges honesty or fitness shall inform the appropriate authority. A judge is immune from civil suits for such reports.

P.

Q.

General Rule re: Disqualification of Judges: A judge shall disqualify himself or herself in a proceeding in

68

which the judge's impartiality might reasonably be questioned. Specific Rules: 1. the judge has a personal bias or prejudice concerning a party or a party's lawyer (but not necessarily an issue), or personal knowledge of disputed evidentiary facts concerning the proceeding. Example: J is assigned to hear the case of P v. D. Three years ago, when J was in private practice, D sued J for malpractice and won. J should recuse himself. Example: J is assigned to hear the capital murder trial of D. In private practice, J was a vocal opponent of capital punishment. J is not required to recuse herself. 2. the judge served as a lawyer in the matter in controversy or a lawyer with whom the judge previously practiced law served during such association as a lawyer concerning the matter, or the judge has been a material witness concerning it Example: J is assigned to hear the case of P v. D. While in private practice, J represented D is this matter. J should recuse himself. Example: J is assigned to hear the case of P v. D. While in private practice in the firm of J & F, Js partner (or associate), F, represented D is this matter. J should recuse himself. The result would be different if Fs representation of D commenced after J left private practice. In other words, a judge should not preside over any cases that were pending in judges former firm while the judge was there. MPRE Question 6 Example: J is a former assistant district attorney. F worked with J at the district attorneys office. While J was at the district attorneys office, F handled the prosecution of D. J left the district attorneys office to take a seat on the bench. The

69

states case against D has been assigned to J. Is J disqualified? Unlike former private practice attorneys, attorneys in government agencies are not considered associated for purposes of this rule. Thus, J is not automatically disqualified under this rule, despite the fact that J was at the district attorneys office at the time F was prosecuting D. J would, of course, be disqualified if he had personally prosecuted Ds case. 3. the judge knows that he or she, individually or as a fiduciary, or the judge's spouse, parent or child (regardless of where the child lives) or any other member of the judge's family residing in the judge's household, has a more than de minimis economic interest in the subject matter in controversy or in a party to the proceeding that could be substantially affected by the proceeding; Example: J is assigned to here the case of X v. ABC Corp. X is suing ABC corp. for $2 million. ABC corp. has a market capitalization of $20 million. J owns 10% of ABCs stock. J should recuse herself. Example: J is assigned to here the case of X v. ABC Corp. X is suing ABC corp. for $2 million. ABC corp. has a market capitalization of $20 million. Js spouse, H, owns 10% of ABCs stock. J should recuse herself. Example: J is assigned to here the case of X v. ABC Corp. X is suing ABC corp. for $25,000. ABC corp. has a market capitalization of $200 million. J owns 2 shares of ABC Corp. stock valued $100. Js interest is de minimis and would not disqualify her. Example: J is assigned to here the case of X v. ABC Corp. X is suing ABC corp. for $25,000. J owns 200 shares of Fidelity Fund, a mutual fund that has many holdings, including ABC Corp. stock (1% of the Fund). Js interest is de minimis (which is the usual case when the economic interest is owned via a mutual fund) and would not disqualify her. Like mutual funds, other indirect investments (e.g., a judges accounts at a

70

bank, securities held by a charity of which the judge is an officer or director, government securities) will not disqualify the judge. 4. the judge or the judge's spouse, or a person within the third degree of relationship (greatgrandparents to great-grandchildren) to either of them, or the spouse of such a person: a. b. c. is a party to the proceeding, or an officer, director or trustee of a party; is acting as a lawyer in the proceeding; is known by the judge to have a more than de minimis interest that could be substantially affected by the proceeding; is to the judge's knowledge likely to be a material witness in the proceeding.

d.

5 The fact that a lawyer in a proceeding is affiliated with a law firm with which a relative of the judge is affiliated does not of itself disqualify the judge. Example: J is assigned to here the case of X v. ABC Corp. X is represented by the Smith Law Firm. Xs trial lawyer is L, a partner at the Smith Law Firm B is also a partner at the Smith Law Firm, but has no involvement with the trial. B is Js son-in-law. J is not automatically disqualified. 6. Remittal: A judge disqualified may disclose on the record the basis of the judge's disqualification and may ask the parties and their lawyers to consider, out of the presence of the judge, whether to waive disqualification. The parties consent must be on the record. Personal bias may not be waived. Rule of Necessity: In some instances, an otherwise disqualified judge may be required to preside over a matter (e.g., a judge might be required to participate in review of a judicial salary statute in which all judges would be interested or might be the only judge available in

7.

71

a matter requiring immediate judicial action, such as a hearing on probable cause or a temporary restraining order. In the latter example, the judge should disclose on the record the basis for possible disqualification and use reasonable efforts to transfer the matter to another judge as soon as practicable). IV. A Judge Shall Avoid Conflicts of Interests A. A judge shall not express bias or prejudice, even outside the judge's judicial activities. Example: improper. B. J tells an ethnic joke to friends. This is

A judge may speak, write, lecture, teach and participate in other extra-judicial activities concerning the law, the legal system, the administration of justice. Example: J teaches as an adjunct professor at the local law school. This is proper, and J may accept the normal salary for such service.

C.

A judge shall not appear at a public hearing before, or otherwise consult with, an executive or legislative body or official except on matters concerning the law, the legal system or the administration of justice, or on personal business. Example: Judge wishes to build a garage next to his house. The local zoning laws prohibit such a building. Judge may appear before the zoning board to request a variance, but may not use his judicial office for influence. Example: Judge is asked to testify before Congress on the new sentencing guidelines. This is appropriate. Example: Judge wishes to testify before Congress in opposition to the war in Iraq. This is inappropriate.

D.

A judge shall not accept appointment to a governmental committee or commission or other governmental position that is concerned with issues of fact or policy on

72

matters other than the improvement of the law, the legal system or the administration of justice. A judge may, however, represent a country, state or locality on ceremonial occasions or in connection with historical, educational or cultural activities. E. A judge may not serve on the board of a public educational institution, but may serve on the board of a public law school or any private educational institution. Example: A judge may serve on the Board of Directors of Yale University (a private college), or Ohio State University School of Law (a public law school), but not Indiana University (a public college). F. A judge may serve as an officer, director, trustee or nonlegal advisor of an organization or governmental agency devoted to the improvement of the law, the legal system or the administration of justice or of an educational, religious, charitable, fraternal or civic organization not conducted for profit, subject to the following limitations and the other requirements of this Code. 1. A judge shall not serve as an officer, director, trustee or non-legal advisor if it is likely that the organization will be engaged in proceedings that would ordinarily come before the judge. In particular, a judge as an officer, director, trustee or non-legal advisor, or as a member or otherwise: a. may assist such an organization in planning fund-raising and may participate in the management and investment of the organization's funds, but shall not personally participate in the solicitation of funds or other fund-raising activities, except that a judge may solicit funds from other judges over whom the judge does not exercise supervisory or appellate authority (i.e., the judge is one of eight district court judges; the judge may solicit

2.

73

funds from the other district court judges, as long as the judge is not the chief judge); b. may make recommendations to public and private fund-granting organizations on projects and programs concerning the law, the legal system or the administration of justice; shall not personally participate in membership solicitation if the solicitation might reasonably be perceived as coercive or if the membership solicitation is essentially a fund-raising mechanism; shall not use or permit the use of the prestige of judicial office for fund-raising or membership solicitation; A judge must not engage in direct, individual solicitation of funds or memberships in person, in writing or by telephone except in the following cases: 1) a judge may solicit for funds or memberships other judges over whom the judge does not exercise supervisory or appellate authority, 2) a judge may solicit other persons for membership in the organizations described above if neither those persons nor persons with whom they are affiliated are likely ever to appear before the court on which the judge serves and 3) a judge who is an officer of such an organization may send a general membership solicitation mailing over the judge's signature (on the organizations letterhead); A judge must not be a speaker or guest of honor at an organization's fund-raising event, but mere attendance at such an event is permissible if otherwise consistent with the CJC.

c.

d.

e.

f.

74

3.

A judge shall not engage in financial and business dealings that a. may reasonably be perceived to exploit the judge's judicial position, or involve the judge in frequent transactions or continuing business relationships with those lawyers or other persons likely to come before the court on which the judge serves. A judge may, subject to the requirements of the CJC, hold and manage investments of the judge and members of the judge's family, including real estate, and engage in other remunerative activity. A judge shall not serve as an officer, director, manager, general partner, advisor or employee of any business entity, except that a judge may, subject to the requirements of the CJC, manage and participate in: i. a business closely held by the judge or members of the judge's family, or a business entity primarily engaged in investment of the financial resources of the judge or members of the judge's family. a judge may be prohibited from participation by other provisions of this Code when, for example, the business entity frequently appears before the judge's court or the participation requires significant time away from judicial duties.

b.

c.

d.

ii.

iii.

e.

A judge shall manage the judge's investments and other financial interests to minimize the number of cases in which

75

the judge is disqualified. As soon as the judge can do so without serious financial detriment, the judge shall divest himself or herself of investments and other financial interests that might require frequent disqualification. f. A judge shall not accept, and shall urge members of the judge's family residing in the judge's household not to accept, a gift, bequest, favor or loan from anyone except for: i. ii. contributions to a judge's campaign for judicial office a gift incident to a public testimonial, books, tapes and other resource materials supplied by publishers on a complimentary basis for official use, or an invitation to the judge and the judge's spouse or guest to attend a bar-related function or an activity devoted to the improvement of the law, the legal system or the administration of justice;

MPRE Question 43 iii. a gift, award or benefit incident to the business, profession or other separate activity of a spouse or other family member of a judge residing in the judge's household, including gifts, awards and benefits for the use of both the spouse or other family member and the judge (as spouse or family member), provided the gift, award or benefit could not reasonably be perceived as intended to influence the judge in the performance of judicial duties; ordinary social hospitality;

iv.

76

Example: L, a law school classmate of Judge, attends a dinner at Judges house and gives Judge a bottle of wine. This is appropriate. v. a gift from a relative or friend, for a special occasion, such as a wedding, anniversary or birthday, if the gift is fairly commensurate with the occasion and the relationship; a gift, bequest, favor or loan from a relative or close personal friend whose appearance or interest in a case would in any event require disqualification Example: Js parents give J a house worth $1 million. This is appropriate, as J would be required to recuse herself in any matter involving her parents, regardless of the gift. vii. a loan from a lending institution in its regular course of business on the same terms generally available to persons who are not judges; a scholarship or fellowship awarded on the same terms and based on the same criteria applied to other applicants; or any other gift, bequest, favor or loan, only if: the donor is not a party or other person who has come or is likely to come or whose interests have come or are likely to come before the judge; and, if its value exceeds $150.00, the judge reports it in the same manner as the judge reports compensation

vi.

viii.

ix.

77

G.

A judge shall not serve as executor, administrator or other personal representative, trustee, guardian, attorney in fact or other fiduciary, except for the estate, trust or person of a member of the judge's family, and then only if such service will not interfere with the proper performance of judicial duties. 1. A judge shall not serve as a fiduciary if it is likely that the judge as a fiduciary will be engaged in proceedings that would ordinarily come before the judge, or if the estate, trust or ward becomes involved in adversary proceedings in the court on which the judge serves or one under its appellate jurisdiction. The same restrictions on financial activities that apply to a judge personally also apply to the judge while acting in a fiduciary capacity. Example: Judge is a trustee of a trust for her father. The trusts principal asset is 1,000 shares of ABC stock. Judge is assigned to hear a case involving ABC. Judge should recuse herself.

2.

H.

A judge shall not act as an arbitrator or mediator or otherwise perform judicial functions in a private capacity unless expressly authorized by law. A judge shall not practice law. Notwithstanding this prohibition, a judge may act pro se and may, without compensation, give legal advice to and draft or review documents for a member of the judge's family. Example: Judge may draft a free will for his parents. Judge may represent himself pro se in a real estate closing.

I.

J.

A judge may receive compensation and reimbursement of expenses for the extra-judicial activities permitted by the CJC (e.g., law school teaching, CLEs), if the source of such payments does not give the appearance of influencing the judge's performance of judicial duties or otherwise give the appearance of impropriety.

78

1.

Compensation shall not exceed a reasonable amount nor shall it exceed what a person who is not a judge would receive for the same activity. Expense reimbursement shall be limited to the actual cost of travel, food and lodging reasonably incurred by the judge and, where appropriate to the occasion, by the judge's spouse or guest. 3. Extra-Judicial compensation and reimbursement must be reported by the judge.

2.

V.

A Judge or Judicial Candidate Shall Refrain From Inappropriate Political Activity A. subject to the exceptions listed in paragraph B below, all judges or candidates (i.e., persons seeking to obtain or retain a judicial office) for election or appointment to judicial office shall not: 1 act as a leader or hold an office in a political organization; publicly endorse or publicly oppose another candidate for public office; make speeches organization; on behalf of a political

2.

3.

4 5.

attend political gatherings; or solicit funds for, pay an assessment to or make a contribution to a political organization or candidate, or purchase tickets for political party dinners or other functions. judges or candidates for election or appointment to judicial office may:

B. 1.

retain during candidacy a public office such as county prosecutor privately express his or her views on judicial candidates or other candidates for public office.

2.

79

C.

A judge shall resign from judicial office upon becoming a candidate for a non-judicial office (except as a delegate to a state constitutional convention) either in a primary or in a general election Example: Judge decides to run for governor. Judge must resign from the bench upon becoming a candidate (i.e., announcing candidacy, filing papers, or authorizing solicitation)

All candidates (judges and lawyers) for judicial office: 1. shall maintain the dignity appropriate to judicial office and act in a manner consistent with the integrity and independence of the judiciary, and shall encourage members of the candidate's family and employees to adhere to the same standards of political conduct in support of the candidate as apply to the candidate; shall not: a. make pledges or promises of conduct in office other than the faithful and impartial performance of the duties of the office; Example: Candidate for judicial offices states: Ill be tough on drug pushers. This is inappropriate. b. make statements that commit or appear to commit the candidate with respect to cases, controversies or issues that are likely to come before the court (this rule does not prohibit a candidate from making pledges or promises respecting improvements in court administration, nor does it prohibit an incumbent judge from making private statements to other judges or court personnel in the performance of judicial duties. Nor does it prohibit the candidate from responding to personal attacks.)

2.

80

Example: Candidate for criminal trial judge tells a reporter: Unlike my opponent, Ill use the death penalty for murderers. This is inappropriate. MPRE Question 16 c. knowingly misrepresent the identity, qualifications, present position or other facts concerning the candidate or an opponent

3.

Candidates for Appointed Positions: A candidate for appointment to judicial office or a judge seeking other governmental office shall not solicit or accept funds, personally or through a committee or otherwise, to support his or her candidacy. a. A candidate for appointment to judicial office or a judge seeking other governmental office shall not engage in any political activity to secure the appointment except that such persons may: i. communicate with the appointing authority, including any selection or nominating commission or other agency designated to screen candidates; seek support or endorsement for the appointment from organizations that regularly make recommendations for reappointment or appointment to the office (e.g., the ABA)

ii.

a non-judge candidate for appointment to judicial office may, in addition, unless otherwise prohibited by law, retain an office in a political organization; attend political gatherings; and continue to pay ordinary assessments and ordinary contributions to a political organization or

81

candidate and purchase tickets for political party dinners or other functions. 4. Elected Positions: A judge or a candidate subject to public election may, except as prohibited by law: a. at any time i. purchase tickets for and attend political gatherings; identify himself or herself as a member of a political party; and contribute to a political organization

ii.

iii. b.

when a candidate for election i. speak to gatherings on his or her own behalf; appear in newspaper, television and other media advertisements supporting his or her candidacy; distribute pamphlets and other promotional campaign literature supporting his or her candidacy; publicly endorse or publicly oppose other candidates for the same judicial office in a public election in which the judge or judicial candidate is running.

ii.

iii.

iv.

c.

A candidate shall not personally solicit or accept campaign contributions or personally solicit publicly stated support. A candidate may, however, establish committees of responsible persons to conduct campaigns for the candidate through media advertisements, brochures,

82

mailings, candidate forums and other means not prohibited by law. Such committees are not prohibited from soliciting and accepting reasonable campaign contributions and public support from lawyers. Such committees may not solicit funds more than one year before the primary election or more than 90 days after the general election. MPRE Questions 27 & 47 d. a candidate for judicial office in a public election may permit the candidate's name: (a) to be listed on election materials along with the names of other candidates for elective public office, and (b) to appear in promotions of the ticket.

5.

Discipline: a judge who violates the campaign rules will be subject to discipline under the CJC; a lawyer-candidate will be subject to discipline under the Rules and the CJC.

83

Вам также может понравиться